Download as pdf or txt
Download as pdf or txt
You are on page 1of 50

3 Sequence and Series

3.1 INTRODUCTION & 3, 6, 12, 24, ..... (Here each term can be obtained
by multiplying the preceding term by 2)
Defined informally, a sequence is a list of numbers.
We are most interested in lists of numbers that satisfy Instead of using f(x) notation to indicate a
some pattern. For example 2, 4, 6, 8, 10, .... is a list of sequence, it is customary to use an, where n represents
the natural number multiples of 2. This can be written an element in the domain of the sequence. Thus, an =
2n, where n is a natural number, so a sequence may f(n). The sequence itself will be denoted by the
be defined like a function by a variable expression. symbol (an) or {an}.
More formally a sequence is defined as follows. The letter n is used instead of x as a reminder that
n represents a natural number. The elements in the
'HILQLWLRQRI6HTXHQFH
range of a sequence, called the terms of the sequence,
A sequence is a function of natural numbers with co- are a1, a2, a3,...., an , .....
domain as the set of real numbers (complex numbers)
If range is a subset of real numbers (complex 7KH JHQHUDO WHUP RU QWK WHUP RI WKH
numbers) then it is called a real sequence (complex VHTXHQFHLVDQ
sequence). A sequence is often given by the formula of its nth
A sequence whose range is a subset of real numbers (or general) term :
is called a UHDOVHTXHQFH. an = f(n) (n = 1, 2, 3, ...)
enabling us to compute a certain term of the
([DPSOHVRI5HDO6HTXHQFH
sequence by its number.
$ 4, 7, 10, 13, ..... (Here each term of the sequence can
The first term is found by letting n = 1, the
be obtained by adding 3 to the preceding term)
second term is found by letting n =2, and so on.
% 5, 3, 1, 1, .... (Here each term of the sequence
For instance, if it is known that an = n2 for any
can be obtained by subtracting 2 from the
n, then a1 = 1, a2 = 4, a3 = 9, and so on
preceding terms)
 Comprehensive Algebra

an = 2n 1 denotes the sequence 1, 2, 3, ..... 1 1 1


1, 2, , 4,....., , 2k, ....
(a sequence of odd integers). 3 2k  1 2k  1
1 1 1
an = denotes the sequence 1, , , .... The factor (1)n in the general
n 2 3
(a harmonic sequence). term of a sequence causes the
signs of the terms to alternate between positive and
The figure shows graphs of negative, depending on whether n is even or odd.
f(x) = 2x and an = 2n. Notice If you start with n = 1 and use consecutive integers,
that f(x) defines a continuous function, while an is the factor (1)n will result in terms with the
discontinuous. alternating signs. + + + etc.
DQ
 
The factor (1)n + 1 will result in terms with the
  alternating signs + + + etc.
 
([DPSOH Write the first four terms of the
   
sequence whose nth term is
3n  2 1 sin nS
(i) (1)n (ii)
DQ Q
5 n2 3
I [  [ 6ROXWLRQ
([DPSOH Write the first five terms for each of 3n  2
(i) Let a n = (1)n
the following sequences: 5
(1) n Putting = 1, 2, 3, 4,...... successively, we get
n 1
(a) an = (b) an = (1)n . n (c) bn = 3 .1  2
n2 2n a 1 = (1)1 =1
6ROXWLRQ 5
(a) Replacing n, in turn, with 1, 2, 3, 4 and 5 gives 3 .2  2
a 2 = (1)2 = 8/5
2, 3, 4, 5,6 5
3 4 5 6 7 3 .3  2
a 3 = (1)3 = 11/5
(b) Replace n with 1, 2, 3, 4 and 5 to get 5
a1 = (1)1 . 1 = 1 3 .4  2
a 4 = (1)4 = 14/5
a2 = (1)2 . 2 = 2 5
a3 = (1)3 . 3 = 3 ..........................................
a4 = (1)5 . 4 = 4 Hence we obtain the sequence
a5 = (1)5 . 1 = 5 1, 8/5, 11/5, 14/5,......
(c) Here, we have b1 = 1/2, b2 = 1/4, b3 = 1/8,
b4 = 1/16, and b5 = 1/32. (ii) Let a n = 12 sin nS
n 3
The formula of the nth term ca n be more
complicated. Putting n = 1, 2, 3, 4,... successively, we get
For instance, the formula
a 1 = 12 sin S  3 / 2
n if n 2k 1 3

an = 1 (k = 1,2......)
n if n 2k  1
a 2 = 12 sin 2 S  3 / 8
specifies the sequence 2 3
Sequence and Series 

Hence, a sequence (an) is defined.


a 3 = 12 sin 3S = 0
3 3 It should be noted that it may not be possible
to represent an by some algebraic formula or by a
a 4 = 12 sin 4 S  3 / 32 recurrence formula.
4 3
........................................... For instance, it is possible to form a sequence of
Hence we obtain the sequence approximate values (with deficit) of the number 2
1.4, 1.41, 1.414, 1.4142,.....
3 /2, 3 /8, 0, 3 /32,... or a sequence of prime numbers (in increasing
order):
3URJUHVVLRQV
2, 3, 5, 7, 11, 13,.....,
It is not necessary that the terms of a sequence always
Although in both cases we do not have the formula
follow a certain pattern or they are described by some
of the nth term or a recurrence formula. For every
explicit formula of the nth term. Those sequences
sequence a rule must be given, enabling us to obtain
whose terms follow certain patterns are called any of its terms. The form in which this rule is given
progressions. is of no importance.
5HFXUVLRQ)RUPXOD  ([DPSOH  Find the first four terms for the
Sequences can also be defined using recursion sequences defined as follows :
formulae. A recursion formula defines the nth term of (a) a1 = 4 ; for n > 1, an = 2. an 1 + 1
a sequence as a function of the previous term(s). (b) a1 = 2; for n > 1, an = an 1 + n 1
A sequence given by a recurrence formula uses the 6ROXWLRQ
knowledge of the previous terms to find its terms. (a) This is an example of a recursive definition.
Since
Those sequences which are defined explicitly,
a1 = 4, and an = 2. an 1 + 1,we have
with a formula for an does not depend on any previous
a2 = 2 . a1 + 1 = 2 . 4 + 1 = 9
term for finding a term.
a3 = 2 . a2 + 1 = 2 . 9 + 1 = 19
When a sequence is specified by a recurrence a4 = 2 . a3 + 1 = 2 . 19 + 1 = 39
formula, the following is usually indicated: (b) a1 = 2, an = an 1 + n 1
$ The first term of the sequence (or first several a2 = a1 + 2 1 = 2 + 1 = 3
terms); a3 = a2 + 3 1 = 3 + 2 = 5
% The formula enabling us to determine any term
a4 = a3 + 4 1 = 5 + 3 = 8
of the sequence from a knowledge of its
previous terms. ([DPSOH A sequence of numbers a1, a2, a3,...
For example, let us consider the sequence (an) satisfies the relation an + 1 = an + a n 1 for n t 2. Find
a4 given a1 = a2 = 1. Note that this sequence is called
whose first term is equal to 1, the second to 2, and
Fibonacci sequence.
each subsequent term, beginning with the third,
6ROXWLRQ Putting n = 2 in the given relation, then
equals the sum of two previous terms: we obtain a3 = a2 + a1 = 1 + 1 = 2
a1 = 1, a2 = 2, an + 2 = an + an + 1 Again putting n = 3, then we obtain
then a4 = a3 + a2  a3 = 2)
a3=1 + 2 = 3, a4 = 2 + 3 = 5, a5 = 3 + 5 = 8, =2+1=3
etc. Hence a4 = 3
 Comprehensive Algebra

([DPSOH A sequence of numbers a1, a2, a3,... If we are given the first few terms of a sequence, the
satisfies the relation a2n + 1 = an an + 2 + (1)n then find best we can do is to find one possible general term,
a3 if a1 = 2 and a2 = 5 while realizing that more than one formula for an
6ROXWLRQ Putting n = 1 in the given relation, we may be correct.
obtain
([DPSOH Find a possible formula for the general
a22 = a1a3 + (1)1
term of the sequences:
52 = 2a3 1 ( a1 = 2, a2 = 5)
2a3 = 26 (i) 3, 5, 7, 9. 11,......
a3 = 13 (ii) 1, 4, 9, 16, 25,.....
(iii) 3, 12, 27, 48, 75,....
)LQGLQJWKHQWK7HUPRID6HTXHQFH 6ROXWLRQ
(i) n : 1, 2, 3, 4, 5, ...
So far we have listed terms in a sequence with the
p p p p p
general term given. In contrast, listing the first few Terms : 3, 5, 7, 9, 11, ...
terms of a sequence is not enough to say for certain p p p p p
what the general term is. However, we can make a 2.1 + 1,2.2 + 12.3 + 12.4 + 12.5 + 1 ...
prediction by looking for a pattern. For example, the One pattern is that each term is 1 more than twice n.
sequence, 1, 4, 9,16 ....... has terms that are squares of A possible formula for the general term is an = 2n + 1
consecutive even integers. (ii) n : 1, 2, 3, 4, 5, ...
p p p p p
Thus, a possible formula for the general term is an = n2.
Terms : 1, 4, 9, 16, 25, ...
It is important to realize that simply listing the first
p p p p p
few terms is not sufficient to define a sequence the (1)1 . 12,(1)2 . 22 (1)3 . 32 (1)4 . 42
nth term must be given. To see this, consider the (1)5 . 52 ...
following sequences, both of which have the same The terms of the sequence are the squares of
first three terms. consecutive positive integers with alternating signs.
A possible formula for the general term is an= (1)nn2.
1 , 1 , 1 , 1 ,..... 1 ,.....
2 4 8 16 2n (iii) n : 1, 2, 3, 4, 5, ...
p p p p p
1 , 1 , 1 , 1 ,....., 6 ,..... Terms : 3, 12, 27, 48, 75, ...
2 4 8 15 ( n  1)(n 2  n  6) p p p p p
When given the first few terms of a sequence, you 3 . 12, 3 . 22, 3 . 32, 3 . 42, 3 . 52, ...
can, however, be asked to find the apparent nth term. One pattern is that each term is 3 times the square of n.
A possible formula for the general term is an = 3n2.
Write the first five terms for
the following sequences When we replace n in the gen-
whose general term is given. eral term by 2n 1, n N we
$ an = n2 generate the odd-positioned terms. Similarly, when
we replace n in the general term by 2n, n N we
S  25 generate the even-positioned terms.
B. an = n2 + (n 1)(n 2)(n 3)(n 4)
1.2.3.4
What do you observe? Can you see why listing the )LQLWHDQG,QILQLWH6HTXHQFHV
first few terms of a sequence is not enough to define A sequence is said to be finite or infinite according
the general term in only one way ? as the number of elements is finite or infinite.
Sequence and Series 

A sequence is a finite sequence if the domain is the FRQYHUJHQW and to converge to that real number.
set {1, 2, 3, 4,...., n}, where n is a natural number. An Graphs of sequences illustrate this property. For
infinite sequence has the set of all natural numbers n 1
instance, the sequence an = is graphed in the
as its domain. n2

The sequence of natural number multiples of 2, figure. We notice that the sequence converges to 1. A
2, 4, 6, 8, 10, 12, 14,....., is infinite, sequence that does not converge to some number is
but the sequence of days in the month of June GLYHUJHQW.
1, 2, 3, 4, ....., 29, 30 is finite. DQ
,QFUHDVLQJDQG 'HFUHDVLQJ6HTXHQFHV C

A sequence (a n) is called increasing if each


subsequent term (or successor) is greater than its
preceding term (or predecessor), i.e., an + 1 > an for
any n.
Examples of increasing sequences :
(i) 1, 4, 9, 16,......... n2, .....; an = n2 ;    Q
(ii) 2, 0, 2, 4,........., 2n 4, .......; an = 2n 4;

(iii) 1 , 2 , 3 , 4 ,......., n ,.....; a n n


2 3 4 5 n 1 n 1 Practice Problems
A sequence (an) is called decreasing if each successor
is less than its predecessor,  Write the sequence whose nth term is
(i) 3n + 1 (ii) n2 + 1
i.e. an + 1 < an for any n.
Examples of decreasing sequences : n n2
(iii) (iv)
n 1 n 1
1 , 1 , 1 ,......., 1 ,.....; a 1;
(i) 1, n  Find the 5th term of the sequence whose first
2 3 4 n n
three terms are 3, 3, 6 and each term after the
(ii) 1, 2, 3, 4, ...., n, .... ; an = n.
second is the sum of two preceding terms.
Increasing and decreasing sequences are called
 The Fibonacci sequence is defined by
monotonic. Not every sequence is monotonic. a1 = a2 = 1 and an = an 1 + an 2, n > 2. Find
For instance, the finite sequence 7, 5, 6, 0, 1,
D Q 
the infinite sequence , for n = 1, 2, 3, 4, 5.
DQ
1, 1, 1, 1, ...., (1)n 1, .... ; an = (1)n 1,
 Write the first five terms of the sequences:
are neither increasing nor decreasing.
n
Note that a sequence whose terms are equal is said 1
to be constant. (i) an= ( n  1) (ii) an=(1)n(2n)
3
For instance, the sequence 5, 5, 5, ....., 5, .... ;
4n  1
an = 5 is constant. (iii) an=
n2  2
&RQYHUJHQWDQG'LYHUJHQW6HTXHQFHV  Decide whether the given sequence is finite or
infinite.
If the terms of an infinite sequence get closer and
(i) The sequence of days of the week
closer to some real number, the sequence is said to be
 Comprehensive Algebra

(ii) 1, 2, 3, 4,..... ([DPSOHV


(iii) a1 = 3 ; for 2 d n d 10, an =3  an 1 (i) 1 + 2 + 3 + 4 + ............................ + n
 Find the first four terms for each sequence : (ii) 2 + 4 + 8 + 16 + ............................
(i) a1 = 2, an = an 1 + 3, for n > 1. A ILQLWHVHULHV is an expression of the form
(ii) a1 = 1, a2 = 1, an = an 1 + an 2, for n t 3 n
Sn = a1 + a2 + a3 + .... + an = 6 a i
 Find the third, the sixth and the eighth terms of t 1
the sequence and LQILQLWHVHULHV is an expression of the form
f
2, where n is an odd number Sn = a1 + a2 + a3 + ..... + an + .... = 6 a i
i 1
an = n  1
n , where n is an even number the letter i is called the index of summation.

 Write the formula of the nth term for each of the Do not confuse this use of i
given sequences : with the use of i to represent
1 , 1 , 1 , 1 ,.....; 3 4 5 an imaginary number. Other letters may be used for
(a) (b) 2, , , ,.....; the index of summation.
2 4 8 16 2 3 4
1 , 1 , 1
8VLQJ6XPPDWLRQ1RWDWLRQ
(c) , ... ;
1.2 2.3 3.4 Polynomial functions, defined by expressions of the
(d) 1, 4, 9, 16, 25,......; form f(x) = anxn + an 1xn 1 + ..... + a1x + a0,
can be written in compact form, using summation
10 , 20 , 30 ,......; 3 5 7 notation, as
(e) (f) , , ,......;
3 9 27 5 7 9 n
f(x) = 6 a i x i
i 0
3.2 SERIES
6
([DPSOH Evaluate the series 6 (2 k  1) .
Suppose a sequence has terms a1, a2, a3, .... Then Sn is k 1
defined as the sum of the first n terms. That is, 6ROXWLRQ Writing each of the six terms, we can
Sn = a1 + a2 + a3 + ....... an . The sum of the first n terms evaluate the sum.
of a sequence is called a series. Special notation is 6
used to represent a series. The symbol 6 , the Greek 6 (2 k  1) = (21 + 1) + (22 + 1) + (23 + 1) + (24 + 1)
k 1
capital letter sigma, is used to indicate a sum. + (25 + 1) + (26 + 1)
n
Thus, the series a1 + a2 + ... + an is abbreviated as 6 a i = (2 + 1) + (4 + 1) + (8 + 1) + (16 + 1) + (32 + 1)
i 1 + (64 + 1)
where i is the index of summation, n is the upper = 3 + 5 + 9 + 17 + 33 + 65 =132.
limit of summation, and 1 is the lower limit of
summation. 8VLQJ6XPPDWLRQ1RWDWLRQZLWK6XEVFULSWV
([DPSOH Write out the terms for each of the
When the term series is used, it refers to following series and evaluate each sum.
the indicated sum not to the sum itself. 3
For example, 1 + 3 + 5 + 7 is a finite (a) 6 (6 x i  2) if x1 = 2, x2 = 4, x3 = 6
i 1
series with four terms. When we use the phrase sum
4
of a series, we will mean the number that results (b) 6 (6 x i  2) f(xi) 'x if f(x) = x2, x1 = 0,
from adding the terms, the sum of the series is 16. i 1

x2 = 2, x3 = 4, x4 = 6 and 'x = 2
Sequence and Series 

6ROXWLRQ = 2n2 + 3n 2(n 1)2 3 (n 1) = 4n + 1


(a) Let i = 1, 2, and 3 respectively, to get so the 50th term is a50 = 201.
3
6 (6 x i  2) = (6x1 2) + (6x2 2) + (6x3 2) 6XPPDWLRQ3URSHUWLHV
i 1
Now, substitute the given values for x1, x2 and x3 Several properties of summation are given below.
3
6 (6 x i  2) = (6 . 2 2) + (6 . 4 2) + (6 . 6 2) These provide useful shortcuts for evaluating series.
i 1
If a1, a2, a3, ..... an and b1, b2, b3, ..... bn are two
= 10 + 22 + 34 = 66
4
sequences and c is a constant, then for every positive
(b) 6 (6x i  2) f(xi) 'x = f(x1)'x + f(x2)'x integer n,
i 1
n
+ f(x3)'x + f(x4)'x
(a) 6 c = nc
= x12 'x + x22 'x + x32 'x + x12 'x i 1

= 02 (2) + 22 (2) + 42 (2) + 62 (2) n n


(b) 6 ca i c 6 ai
= 0 + 8 + 32 + 72 = 112 i 1 i 1
n n n
([DPSOH Use summation notation to rewrite (c) 6 (a i  b i ) 6 a i  6 bi
i 1 i 1 i 1
each series with the index of summation starting at
n n n
the indicated number : (d) 6 (a i  b i ) 6 ai  6 bi
8 i 1 i 1 i 1
10
(a) 6 (3i  4 ); 0 (b) 6 i 2 ;  1 To prove Property (a), expand the series to get
i 1 i 2
6ROXWLRQ (a) Let the new index be j. Since the c + c + c + c + .... + c,
new index is to start at 0, which is j = i 1, or i = j + 1. where there are n terms of c, so the sum is nc.
Substitute j + 1 for i in the summation. Property (c) also can be proved by first expanding
j 1 8 j 7 the series
8
n
6 (3i  4); 0 = 6 [3( j  1)  4 ] = j60 (3 j  1) 6 (a i  b i ) = (a1 + b1) + (a2 + b2) + ..... + (an + bn).
i 1 j 1 1
i 1
7 Now, use the commutative and associative properties
or 6 (3 j  1) to rearrange the terms.
j 0
n
(b) Here, if the new index is j, then i = j + 3 and
j 7
6 (a i  b i ) =(a1 + a2 + ...... + an) + (b1 + b2 + ... + bn)
10 j 3 10 7 i 1
2 2 2
6 i 2 = 6 ( j  3) = 6 ( j  3) or j 61( j  3) n n
i 2 j 3 2 j 1
= 6 a i  6 bi
i 1 i 1
)LQGLQJ*HQHUDO7HUPIURP6XPPDWLRQ
Proofs of the other two properties are similar.
)RUPXOD
([DPSOH Use the properties of series to evaluate
We have Sn = a1 + a2 + a3 + ..... + an 6
Also Sn 1 = a1 + a2 + a3 + ..... + an 1 6 (i 2  3i  5) .
i 1
Hence, an = Sn Sn 1 6 6 6 6
6ROXWLRQ 6 (i 2  3i  5) = 6 i 2  6 3i  6 5
([DPSOH If the sum of n terms of a series is i 1 i 1 i 1 i 1
6 6 6
given by 2
= 6 i 36 i 65
Sn = 2n2 + 3n, find its 50th term. i 1 i 1 i 1
6 6
6ROXWLRQ Let an be the nth term of the series so an 2
= 6 i  3 6 i  6(5)
= Sn Sn 1. i 1 i 1
 Comprehensive Algebra

6(6  1)( 2.6  1) 6(6  1) its first three terms, etc., as the 1st, 2nd, 3rd, ..... terms
=  3  6(5) of the sequence. Thus
6 2 S1 = a1
= 91 + 3(21) + 6(5) = 184 S2 = a1 + a2
S3 = a1 + a2 + a3
:ULWLQJ6XPVLQ6XPPDWLRQ1RWDWLRQ .................................................
 ([DPSOH  Write the sum using summation Sn = a1 + a2 + a3 + ..... + an
.................................................
notation
(i) 1 + 8 + 27 + 64 + 125 + 216 + 343 The infinite series, an is said to be convergent,
(ii) 1 + 8 27 + 64 125 + 216 343 divergent or oscillatory according as (S n ) is
(iii) 3 + 9 + 27 + 81 + ... convergent, divergent or oscillatory.
6ROXWLRQ (i) 1 + 8 + 27 + 64 + 125 + 216 + 343 If in the case of convergence, Sn os as n o f ,
This is the sum of cubes, 13 + 23 + 33 + 43 + 53 + 63 + 73.
Using i for the index of summation, a possible general where s is finite, we say that an is convergent
term is i3. Thus,
7 and its sum is equal to s.
1 + 8 + 27 + 64 + 125 + 216 + 343 = 6 i 3
(ii) 1 + 8 27 + 64 125 + 216 343
i 1 If Sn of as n of, we say that an is divergent
and diverges to f.
This is almost like the series in part (a) except
If Sn o f as n o f, we say that the series is
that the signs alternate. Since (1)i = 1 when i divergent and diverges to f.
is even and (1)i = 1 when i is odd, we write The series is said to oscillate finitely and infinitely
the series in summation notation as follows : according as (Sn) oscillates finitely or infinitely.

7 Let us consider a r 1 .
1 + 8 27 + 64 125 + 216 343 = 6 (1) i i 3 r( r  1)
i 1
(iii) 3 + 9 + 27 + 81 + ..... Here Sn 1  1  1  .....  1 .
This infinite series consists of powers of 3. We 1.2 2.3 3.4 n( n  1)
use the symbol f to represent infinity and write
We further note that a r 1 1 1 .
the series in summation notation as follows :
r( r  1) r r  1
f
Hence putting r = 1, 2, ..... , n we have
3 + 9 + 27 + 81 + ...... = 6 3i
i 1 1
a1 = 1
,QILQLWH6HULHV 2
1 1
The expression a1 + a2 + a3 + ..... + an + ..... is called a2 =
2 3
f
an infinite series. It is, in short, written as 6 a i 1 1
i 1 a3 =
3 4
.......................
where is the notation of summation. 1 1
an
We deduce a sequence (Sn), called the sequence n n 1

of partial sums, from the series an by taking its Adding, we get Sn = 1 1


n 1
first term, the sum of its first two terms, the sum of
Sequence and Series 

Therefore, Sn o1 as n o f.  Use summation notation to rewrite each series


The series is, therefore, convergent and its sum is 1. with the index of summation starting at the
f indicated number:
([DPSOH Evaluate 6 a i if ar = r..
i 1 5 10
6ROXWLRQ Here Sn = 1 + 2 + 3 + .... + n = (i) 6 (6  3i) ; 3 (ii) 6 2(3) i ; 0
i 1 i 1
1
n (n+ 1). 9
2 (iii) 6 ( i 2  2i) ; 0
Obviously, Sn o f as n o f. i 1

Hence 6r is divergent and diverges to f.  Evaluate


If ar = r, it can be easily shown that 6 a n is 5 4
divergent and diverges to f. (i) 6 ( 4i 2  2i  6) (ii) 6 (3i 3  2 i  4)
i 1 i 1
f
([DPSOH Evaluate 6 a i if ar = (1)r1.  Write each series in summation notation. Use
i 1
6ROXWLRQ Here Sn = 1 1 + 1 1 + 1 ..... to the index i and let i begin at 1.
n terms. 1 1 1 1
(i) 1 + + ....
Therefore, Sn = 0 or 1 according as n is even or odd. 2 4 8 128
Hence Sn o 0 or 1 as n o f. x  x  x  x
(ii)
x 1 x  2 x  3 x  4
Moreover |Sn| d 1. (iii) 8 + 16 + 24 + 32 + 40 + ...
Thus (Sn) oscillates finitely.
 If f is a function satisfying f(x + y) = f(x) f(y) for
Therefore, 6an is oscillatory and oscillates finitely.
n
f all x, y N such that f(1) = 3 and 6 f ( x) =
([DPSOH Evaluate 6 a i if ar = (1)r1r.. x 1
i 1
6ROXWLRQ Here Sn = 1 2 + 3 4 + 5 6 + ..... 120, find the value of n.
to n terms.
n 1
Therefore, Sn = or (n + 1) according as n is
3.3 ARITHMETIC PROGRESSION
2 2 (A.P.)
even or odd.
Thus Sn ofor f according as n is even or odd. A sequence of numbers each term of which,
Hence (Sn) oscillates infinitely. beginning with the second, is formed by adding a
constant number to the predecessor is called an
arithmetic progression.
Practice Problems
If a sequence (an) is an arithmetic progression, then,
 Evaluate each series : by definition,
4 4 6 a2 a1 = a3 a2 = .... = an + 1 an = ...,
(i) 6 1 (ii) 6 i i (iii) 6 ( 1) k . k
j 1 j i 1 k 1 that is, the difference between any term and its
predecessor is equal to one and the same number.
 Evaluate the terms for each sum, where
This constant number is called the FRPPRQ
x1 = 2, x2 = 1, x3 = 0, x4 = 1 and x5 = 2 :
GLIIHUHQFH of an arithmetic progression and is
denoted by d.
5 5 xi  1
(i) 6 (2x i  3) (ii) i62
i 1 xi  2 Thus, an arithmetic progression (an) is defined by
the following conditions :
 Comprehensive Algebra

(i) a1 = a, where a is called the first term Adding these equalities termwise, we obtain
(ii) Dn + 1 = an + d for any n t 1. 2Sn = (a1 + an) + (a2 + an 1) + (a3 + an 2)
+ ...... + (an 1 + a2) + (an + a1)
If a is the first term and d the common difference,
then AP can be written as On the right hand side of the equality the sum of
a , a + d , a + 2 d , ....... a + (n  1) d , ....... . two numbers in each parentheses is equal to a1 +
an. Indeed,
Each of the following series forms an arithmetical a2 + an 1 = (a1 + d) + (an d) = a1 + an,
progression : a3 + an 2 = (a2 + d) + (an 1 d) = a2 + an 1
3, 7, 11, 15,.... = a1 + an, etc.
8, 2, 4, 10,.....
The number of terms in the parentheses is equal to
p, p q, p 2q, p 3q,......
n. Therefore,
The common difference is found by substracting any n(a1  a n )
term of the series from that which follows it. In the first 2Sn = (a1 + an)n, Sn = ,
2
of the above examples the common difference is 4; in
which is the formula for the sum of n terms of an
the second it is 6; in the third it is q.
arithmetic progression.
Let (an) be an arithmetic progression, a1 its first term,
Replacing in this formula the terms a n by the
and d its common difference.
expression a1 + d(n 1), we obtain
Let us derive the formula for the nth term of an
n
arithmetic progression. Sn = 2a 1  ( n  1)d
By the definition of the arithmetic progression, 2
a2 = a1 + d, This formula expresses the sum of n terms of an
a3 = a2 + d, arithmetic progression (an) in terms of the first term,
a4 = a3 + d, common difference and the number of terms.
....................... If a is the first term, n be the number of terms, and
an 1 = an 2 + d, if " denotes the last, or nth term, we have
an = an 1 + d.
" = a + (n 1) d.
Adding these n 1 equalities termwise, we obtain n
(a2 + a3 + a4 + ..... + an 1) + an ? Sn =(a + " )
2
= a1 + (a2 + a3 + ..... + an 2 + an 1) + (n 1)d,
n
an = a1 + (n 1)d. and Sn = {2a + (n 1) d}
2
This formula makes it possible to find any term of Here we have three useful formulae ; in each of these
an arithmetic progression if its first term and any one of the letters may denote the unknown
common difference are known. Therefore it is called quantity when the three others are known.
the formula of the general term of an arithmetic
progression. Let us now derive the formula for the
sum of (t he first) n terms of an a rithmetic
progression.
Denoting the sum of n terms of an arithmetic (i) ,ID, b , c are in A.P. 2 b = a + c.
progression (an) by Sn, we write this sum twice, "a
(ii) The common difference of an A.P. =
arranging the terms in the second line in a reverse n 1
order : where a = first term of A.P., " = last term of A.P..
Sn = a1 + a2 + a3 + ..... + an 1 + an,
and n = number of terms of A.P.
Sn = an + an 1 + an 2 + ..... + a2 + a1,
Sequence and Series 

The common difference can be zero, positive Further if


or negative. Sn = pn2 + qn, then an = Sn Sn 1
= pn2 + qn {p(n1)2 + q(n1)}= 2pn + (q p)
(iii) Three numbers in A.P. can be taken as a d, a,
which is a linear expression in n and hence the
a + d; five numbers in A.P. can be taken as
sequence is in A.P. Note that common
a 2d, a d, a, a + d, a + 2d.
difference is twice the coefficient of n2.
In general : (2r + 1) numbers in A.P. can be
taken as (r N) (viii) If it is to be determined whether a set of given
a rd, a (r 1) d, ..., a d, a, a + d,...., a + (r 1)d, numbers are terms of an A.P. then let these
a + rd numbers be the pth, qth and rth terms of an A.P.
Then find a relation in p, q, r. If the relation in
(iv) Four numbers in A.P. can be taken as
p,q, r is true for distinct positive integral values
a 3d, a d, a + d, a + 3d; six numbers in A.P.
of p, q, r, then those numbers are terms of an
can be taken as a 5d, a 3d, a d, a + d,
A.P.
a + 3d, a + 5d.
In general : 2r numbers in A.P. can be taken as (ix) If number of identical terms in two A.P.s is to
(r N) be found, then let r be the number of identical
terms in the two A.P.s. Form the sequence of
a (2r 1) d, a (2r 3) d,...., a 3d, a d,
identical terms, it will be an A.P. Find the rth
a + d, a + 3d,....,a + (2r 3) d, a +(2r 1) d
term of this A.P. make tr d the smaller of the
(v) If nth term of any sequence is linear expression last term of the two A.P.s and find the positive
in n then the sequence is in A.P. integral value of r.
Since an = a1 + (n 1)d we have an = d n + (a1 ([DPSOH Is 55 a term of the sequence 1, 3, 5, 7,...
d), which is a linear expression in n. ? If yes, find which term is it.
6ROXWLRQ If possible let nth term of the sequence
Further if an = pn + q, then
be 55.
an an 1 = pn + q {p(n1) + q} = p (constant)
Now tn = a + (n 1)d. Here tn = 55, a = 1, d = 2
Note that the coefficient of n represents the ? 55 = 1 + (n 1)2 or 2n = 56 ? n = 28
common difference. Hence 55 is 28th term of the given sequence.
n If n does not come out to be an integer,
(vi) Any sum of the form 6 (mi  p ) , where m and
i 1 then 55 will not be term of the given
p are real numbers, represents the sum of the sequence.
terms of an arithmetic sequence having first ([DPSOH Suppose that an arithmetic sequence
term a1 = m + p and common difference d = m. has a8 = 16 and a16 = 40, find a1
(vii) If sum of n terms of any sequence is a quadratic 6ROXWLRQ We must find d first. Since
expression in n without the constant term a8 = a1 + (8 1)d,
(i.e., Sn = pn2 + qn), then the sequence is in A.P. replacing a8 with 16 gives
n 16 = a1 + 7d or a1 = 16 7d.
Since Sn = {2a + (n 1) d} = d n2 +
2 2 Similarly, 40 = a1 + 15d or a1 = 40 15d. From
these two equations, we get
(a d )n, we observe that Sn is a quadratic
2 16 7d = 40 15 d d = 3.
expression in n without the constant term. To find a1, substitute 3 for d in 16 = a1 + 7d
16 = a1 + 7(3) a1 = 5
 Comprehensive Algebra

([DPSOH A sequence is given by the formula b2  a 2


=
of its nth term: an = 10 3n. prove that an is an (a  c )( b  c)(a  b )
arithmetic progression. 1  1 = cb
6ROXWLRQ We find an +1 = 10 3(n + 1) = 7 3n a  b a  c (a  b )(a  c)
and consider the difference
an + 1 an = 7 3n (103n) = 3 and d = 3. = c2  b 2
([DPSOH If nth term of a sequence is 2n2 + 1, (a  b)(a  c)( b  c)
find the sequence. Is this sequence an A.P. ? Hence it follows that
6ROXWLRQ Given tn = 2n2 + 1 1  1 1  1
Putting n = 1, we get t1 = 2.12 + 1 = 3 = and
ac bc ab ac
Putting n = 2, we get t2 = 2.22 + 1 = 9
1 1 1
Putting n = 3, we get t3 = 2.32 + 1 = 19 consequently, the numbers , ,
bc ac ab
Putting n = 4, we get t4 = 2.42 + 1 = 33
form an arithmetic progression.
and so on
([DPSOH Prove that if the numbers logkx, logmx
Hence given sequence is 3, 9, 19, 33.... and lognx (x z 1) form an arithmetic progression then
tn 1 = 2(n 1)2 + 1
n2 = (kn) log k m .
From (i) and (ii), we get 6ROXWLRQ 2 logmx = lognx + logkx
tn tn 1 = 2n2 + 1 {2(n 1)2 + 1)
2 1  1 and consequently,,
= 2n2 + 1 {2(n2 2n + 1) + 1} = 4n 2,
log x m log x n log x k
which is not independent of n i.e. it is not a constant,
hence the given sequence is not an A.P. log x m log x m
2= 
([DPSOH Let an = n2 + 1 and bn is defined
log x n log x k
bn = a n + 1 a n . 2 = lognm + logkm
Show that {bn} is an arithmetic sequence. Let us rewrite this equality as
6ROXWLRQ The first formula determine the series :
2 + 5 + 10 + 17 + 26 + ...
lognn2 = lognm + logn n logk m
The second determines the series : Now, raising we obtain
(5 2) + (10 5) + (17 10) + ..... n2 = mn log k m or n2 = ( ( kn ) logk m
or 3 + 5 + 7 + 9 + .............. which is clearly an A.P.
Alternatively, ([DPSOH For what values of the parameter a are
bn = an + 1 an = [(n + 1)2 + 1] (n2 + 1) = 2n + 1 there real values of x such that
which is a linear polynomial in n. a
51+x + 51 x, , 25x + 25x
([DPSOH  If the numbers a2, b2, c2 form an 2
are three consecutive terms of an AP?
arithmetic progression prove that the numbers
6ROXWLRQ We should have
1 1 1 a
, , also form an arithmetic 2. = (51 + x + 51 x) + (25x + 25x)
bc ac ab 2
progression. a = 5(5x + 5x) + (52x + 52x)2 = 5{(5x/2 5x/2)2
+ 2. 5x/2 . 5x/2} + {(5 x 5x)2 + 2.5x . 5x}
6ROXWLRQ Given b2 a2 = c2 b2, Consider
= 5{(5x/2 5x/2)2 + 2} + (5x 5x)2 + 2
1  1 ba
= ? a t 5.2 + 2 = 12
a  c b  c (a  c)( b  c )
Sequence and Series 

([DPSOH Show that cube roots of three distinct or 2E2 = 293 243 = 50or E2 = 25
prime numbers cannot be three terms (not necessarily ? E=5
consecutive) of an A.P. If E = 5, the three numbers are 14, 9, 4
6ROXWLRQ If possible let p1/3 q1/3, r1/3 be three If E = 5, the three numbers are 4, 9, 14.
terms of an A.P. where p, q, r are three distinct prime ([DPSOH Find four numbers in A.P. such that
numbers. their sum is 50 and the greatest of them is 4 times the
Then p1/3 = a, q1/3 = a + md, r1/3 = a + nd; where m, n least.
must be positive integers. On subtracting these 6ROXWLRQ Let the four numbers in A.P. be
equations pairwise and dividing, we get D 3E, D E , D + E, D + 3E.
Given, D 3E + D E + D + E + D + 3E = 50
m q1 / 3  p1 / 3
= 1/ 3 mr1/3 nq1/3 = (m n)p1/3
n r  p1 / 3 25
or 4D = 50 ?D=
(mr1/3 nq1/3)3 = (m n)3 p 2
m3r n3q 3mn r1/3q1/3 (m n)p1/3 = (m n)3p and D + 3E = 4(D 3E) or 3D = 15 E
3 3 3
m r  n q  (m  n ) p D 25 5
p1/3q1/3r1/3 = or E=
3mn (m  n ) 5 5u2 2
irrational number = rational number (  p, q, r Hence the four numbers are 5, 10, 15, 20.
are distinct prime, p1/3q1/3r1/3 cannot become ([DPSOH How many terms are identical in the
rational) which is a contradiction. two arithmetic progressions 2, 4, 6, 8,.... up to 100
([DPSOH When we divide the ninth term of an terms and 3, 6, 9,... up to 80 terms.
arithmetic progression by its second term, we get 5 6ROXWLRQ Let r terms be identical.
as a quotient, and when we divide the thirteenth term Now the sequence of identical terms is 6, 12, 18, ...
of that progression by the sixth term, we get 2 as a Its rth term = 6 + (r 1)6 = 6r
quotient and 5 as a remainder. Find the first term and The 100th term of the sequence 2, 4, 6, 8, ....
the common difference. = 2 + (100 1)2 = 200
6ROXWLRQ From the given conditions :
and 80th term of the sequence 3, 6, 9, .....
a9 = a2 . 5  a1 + 8d = (a1 + d) . 5,
= 3 + (80 1)3 = 240
a13 = 2a6 + 5  a1 + 12 d = 2(a1 + 5d) + 5
containing only two unknowns, a1 and d whose is Since, last term i.e. rth term of the sequence of
a1 = 3, d = 4. identical terms cannot be greater than 200.

([DPSOH The sum of three numbers in A.P. is ? 6r d 200 or r d 200 or r d 33 1


6 3
27 and the sum of their squares is 293. Find the
numbers. ? r = 33. Hence 33 terms are identical.
6ROXWLRQ Let the three numbers in A.P. be DED
DE. If we get r d k, then r = k, if k is an integer;
Given, DEDDE = 27 and r = integer just less than k if k is not
an integer.
or 3D = 27 ?D=9
and (D E)2 + D2 + (D + E)2 = 293
([DPSOH (a) Evaluate S12 for the arithmetic
or D2 + E2 2DE + D2 + D2 + E2 + 2DE = 293. sequence
or 3D2 + 2E2 = 293 or 3.92 + 2E2 = 293 9, 5, 1, 3, 7 ,......
 Comprehensive Algebra

(b) Use a formula for Sn to evaluate the sum of the Thus terms of the given series are 8, 18, 28.... which
first 60 positive integers. are in A.P. whose c.d. is 10.
6ROXWLRQ
(a) We want the sum of the first 12 terms. Using ([DPSOH Find 1 3 + 5 7 + 9 11 + .... to n
a1 = 9, n = 12, and d = 4 in the formula terms.
n [ 2a  ( n  1)d ] 6ROXWLRQ Here alternate terms of the series are in
Sn = 1 A.P.
2
12 &DVH,  When n is even, let n = 2m
S12 = [2( 9)  11( 4)] = 156 1 3 + 5 7 + 9 11 + ... to n terms.
2
= 1 3 + 5 7 + 9 11 + ... to 2m terms.
(b) Here n = 60, a1 = 1 and a60 = 60,
so it is convenient to use the formula = [1 + 5 + 9 + ... to m terms] [3 + 7 + 11 + ...
to m terms]
n (a  a )
Sn = 1 n
m m
2 = {2.1 + (m 1)4} {2.3 + (m 1)4}
60 (1  60) 2 2
S60 = = 1830 = m (2m 1 2m 1) = 2m = n
2
([DPSOH Find the sum of n terms of the series &DVH,,  When n is odd, let n = 2m + 1
1 3 + 5 7 + 9 11 + ... to n terms.
2a 2  1 6a 2  5
, 4a  3 , , ..... = 1 3 + 5 7 + 9 11 + ... to 2m + 1 terms.
a a a
= [1 + 5 + 9 + ... to (m + 1) terms] [3 + 7 + 11
2a 2  1
6ROXWLRQ u1 , u 2 4a  3 . + ... to m terms]
a a
? Common difference m 1 m
= {2.1+ (m + 1 1)4} {2.3 + (m 1)4}
2 2
2
4a  3  2a  1 2 a  1 2 a  1 . m 1 m
= (4m + 2) (4m + 2)
a a a a 2 2
2 2
n 2 . 2a  1  ( n  1) 2(a  1) = (2m + 1)(m + 1 m) = 2m + 1 = n
? Sn
2 a a [ n = 2m + 1]

([DPSOH If the sum of n terms of a series be 6HFRQG0HWKRG:


5n2 + 3n, find its nth term. Are the terms of this series &DVH, When n = 2m
in A.P.? 1 3 + 5 7 + 9 11 + ....... to n terms.
6ROXWLRQ Given Sn = 5n2 + 3n. = 1 3 + 5 7 + 9 11 + ..... to 2m terms.
? Sn - 1=5(n1)2 + 3 (n1) [Putting n1 in place of n] = (1 3) + (5 7) + (9 11) + .... to m brackets
 Sn = (t1 + t2 + ... + tn 1 ) + tn = Sn 1 + tn = 2 2 2 ... to m terms = 2m = n[ n= 2m]
? tn = Sn Sn 1 &DVH,, When n = 2m + 1
= 5n2 + 3n 5 (n 1)2 3(n 1) 1 3 + 5 7 + 9 11 + ...... to n terms
= 5[n2 (n 1)2] + 3[n n + 1] = 1 3 + 5 7 + 9 11 + .... to (2m + 1) terms.
= 5(2n 1) + 3 = 10n 2 = 1 [3 5 + 7 9 + ...... to 2m terms]
? t1 = 10. 1 2 = 8 [Putting n = 1] = 1 [(3 5) + (7 9) + ..... to m brackets]
t2 = 10. 2 2 = 18 [Putting n = 2] = 1 [2 2 ... to m terms] = 1 (2m) = 2m + 1= n
t3 = 10. 1 2 = 28 [Putting n = 3]
and so on.
Sequence and Series 

([DPSOH  Let a n be the nth term of an T24 a  23d


arithmetic progression. Let Sn be the sum of the first ? Tc24 ac  23dc ...(2)
n terms of the arithmetic progression with a1 = 1 and
a3 = 3a8. Find the largest possible value of Sn. Comparing (1) and (2), we get
6ROXWLRQ From a 3 = 3a 8 we obtain 1 + 2d = ? n = 47
3(1 + 7d) a  23d 3 u 47  13 1
from (1), =
ac  23d 3 u 47  21 2
2
d = 19 . T24 1
? T c24 2 T24 : Tc24 = 1 : 2
n 2
Then Sn = 2  ( n  1)
2 19 ([DPSOH Suppose a 1, a 2,.... are in A.P. and
n (20  n )n Sk denotes the sum of the first k terms of this A.P.
= [19 (n 1)] = .
19 19 If Sn/Sm = n4/m 4 for all m, n, 1, then prove that
2 2
Now consider 20nn = [n 20n]
a m +1 (2m + 1)3
= [(n10)2 100] =
a n +1 (2n + 1)3
100  ( n  10)2
? Sn = 6ROXWLRQ Putting a 1 = a, we have
19
now, Sn will be maximum if n = 10 and Sn n[2a + (n - 1)d] / 2 n 4
100 = =
(Sn)max = Sm m[2a + (n - 1)d] / 2 m 4
19
([DPSOH The sum of n terms of two A.P.'s are 2a + (n - 1)d n3
= 3
in the ratio (3n 13) : (5n + 21). Find the ratio of 2a + (m - 1)d m
their 24th terms.
Replacing n by 2n + 1 and m by 2m + 1, we get
6ROXWLRQ Let the two A.P.s be a, a + d,a + 2d,
....and ac, ac + dc, ac + 2dc,...... 2a  (2 n  1  1)d (2n  1)3
Let their sum be Sn and Scn respectively. 2a  (m  1  1)d (2m  1)3
n
? Sn = [2a + (n 1) d] a n 1 (2 n  1)3 a (2m  1)3
2 3
m 1
n
a m 1 (2m  1) a n 1 (2n  1)3
and Scn = [2ac+ (n 1) dc]
2
Sn 3n  13 Practice Problems

Scn 5n  21
n [ 2a  ( n  1)d]  Can the numbers 2, 6 , 4.5 be terms of an
? 2 3n  13
arithmetic progression?
n [ 2ac  ( n  1)dc] 5n  21
2  The lengths of the sides of a quadrilateral form
an arithmetic progression. Can a circle be
a  n  1 d inscribed into the quadrilateral?
2 3n  13
...(1)  Divide 20 into four parts which are in A.P. such
ac  n  1 3 n  21
dc that the product of the first and fourth is to the
2
product of the second and third is 2 : 3.
The 24th terms of two A.P.'s are respectively,
 The sum of four integers in A.P. is 24, and their
T24 = a + 23 d and Tc24 = ac + 23dc
product is 945; find them.
 Comprehensive Algebra

 If length of sides of a right angled triangle are 3URRI


in A.P., show that their ratio is 3 : 4 : 5. By the definition of the arithmetic progression,
 Sum 6 , 3 3 , 12 ,.... to 50 terms. an + 1 = an + d ; an + 2 = an +1 + d,
3 3 an + 1 an = an + 2 an + 1
 Solve a n  a n 2
log x  log 4 3 x  log 6 3 x  ....  log16 3 x Hence, an + 1 =
3 2
= 36 The converse is also true : if a sequence is such
 The sum of 15 terms of an A.P. is 600, and the that any of its terms, beginning with the second, is
common difference is 5; find the first term. an arithmetic mean of the predecessor and successor,
then this sequence is an arithmetic progression.
 Find the arithmetic progression if Indeed, let for any three neighbouring terms of a
1 certain sequence (an) the following relationship be
a1 = 1 and S5 = (S10  S5 )
4 fulfilled :
 The third term of an A.P. is 18, and the seventh a n  a n 2
term is 30; find the sum of 17 terms. an + 1 =
2
 How many terms of the arithmetic progression
Then 2an + 1 = an + an + 2, or an + 1 an = an + 2 an + 1,
18, 16, 14,... should be taken so that their sum
be equal to zero? that is, the difference between any term of the
sequence (an) and its predecessor is equal to one and
 How many terms of the series 15 + 12 + 9 + ....
the same number. Hence (a n) is an arithmetic
must be taken to make 15 ? Explain the double
answer. progression.

 The sum of n terms of a series is 2n2 + 3n. Is the Thus, the established property is inherent in an
series arithmetic? If so, find it . arithmetic progression and only in it.
 If the sum of n terms of an A.P. is 2n + 3n2, find ,PSRUWDQW3URSHUWLHVRI$3
the rth term.
L If each term of an A.P. is increased, decreased,
 Find the sequence in which the sum of any multiplied or divided by the same non-zero
number of terms, beginning with the first, is number, then the resulting sequence is also
four times as large as the square of the number an AP.
of terms.
If a1, a2, a3, ........................ are inA.P. then
 Find a three digit number which is divisible by
(a) a1 k , a2 k, a3 k ............ are in A.P..
45 and whose digits are terms of an arithmetic
(b) a1k , a2k, a3k .............. are in A.P.
progression.
 The sum of n terms of two arithmetic series are a1 a 2 a 3
(c) , , .............. are inA.P..
in the ratio of 7n + 1 : 4n + 27; find the ratio of k k k
their 11th terms. (d) ap , ap + q, ap + 2q .............. are in A.P.
(e) a1r , a 2r , a3r , ...., a rn are not in A.P. if r z
3.4 PROPERTIES OF A.P.
LL The sum of the terms of an AP equidistant
An arithmetic progression (a n) possesses the from the beginning and the end is constant
following characteristic property : any of its terms, and equal to the sum of the first and last
beginning with the second, is an arithmetic mean
terms. If
of the predecessor and successor.
Sequence and Series 

a1, a2, a3,..., an are in A.P., then b+c-a c+a-b a+b-c


a1 + an = a2 + an 1 = a3 + an 2 = ......  If , , are in A.P..
a b c
LLL Any term of an AP (except the first) is equal 1 1 1
show that , , are in A.P. provided a + b
to half the sum of terms which are equidistant a b c
from it. If a1, a2, a3,..., an are in A.P., then +cz0
a rk  a r k  Prove that for any arithmetic progression the
ar =  k, 0 d k d n r following equalities hold true :
2
(i) a1 2a2 + a3 = 0
LY If a1, a2, a3 ............... and b1, b2, b3,......... are two (ii) a1 3a2 + 3a3 a4 = 0
A.P.s, then (iii) a1 4a2 + 6a3 4a4 + a5 = 0
(a) a1 r b1 , a2 r b2 , a3 r b3 ............ are in A.P..
 Prove that for the terms of an arithmetic
(b) a1b1, a2b2, a3b3,.......... are not in A.P. progression the equality ap + am = ap + k + am k
If c1 and d1 be the common difference of the is valid.
two sequences respectively, then for the new  Find the arithmetic progression if it is known
sequence, we have that a1 + a3 + a5 = 12, a1a2a3 = 80.
tn = {a1 + (n 1)c1}{b1 + (n 1)d1}  The sum of three numbers which are consecutive
= a1b1 + (a1d1 + b1c1)(n 1) + c1d1(n 1)2 terms of an arithmetic progression is 2, and the
which is not a linear expression in n, and hence, sum of the squares of those numbers is 14/9.
is not an A.P. Find the numbers.
a1 a 2 a 3  The first term of an A.P. a1, a2, a3,....., an is unity.
(c) , , ,...... are also not in A.P..
b1 b 2 b3 For what value of the common difference is
a1a3 + a2a3 minimum ?
([DPSOH The successive terms of an A.P. are a1,
a2, a3,...... Find S20 if a6 + a9 + a12 + a15 = 20.  Suppose a1, a2, ........, an are in A.P. with first
6ROXWLRQ Noting that a6 and a15 are equidistant term a1 = 0 and common difference d z 0. Show
from the ends also a9 and a12 and that the sum of 2 that
terms equidistant from the ends is the same, we have a3 a4 a 5 an
20 + + + .......
a6 + a15 = a9 + a12 = =10 a2 a3 a 4 a n 1
2
? a1 + a20 = 10 1 1  .......  1 a n 1 a2
d  =
+
20 a 2 a3 a n 2 a2 a n 1
? S20 = (a + l) = 10(a1 + a20) = 10 10 = 100
2
3.5 GEOMETRIC PROGRESSION (GP)
Practice Problems
A number sequence whose first term is different from
 If a, b, c be in A.P., prove that zero and each term, beginning with the second, is
equal to its predecessor multiplied by one and the
1,1,1
(i) are in A.P.. same number is called a geometric progression.
bc ca ab
(ii) b + c, c + a, a + b are in A.P. If a sequence (an) is a geometric progression, then,
(iii) a2 (b + c), b2 (c + a), c2 (a + b) are in A.P. by definition,
ab  ac , bc  ba , ca  bc a2 a3 an a n 1
(iv) are in A.P.. = = ..... = =
bc ca ab a1 a2 a n 1 a n = .....,
 Comprehensive Algebra

that is, the ratio of any term to its predecessor is a3 = a2r,


equal to one and the same number. This number is a4 = a3r,
called the common ratio of a geometric progression ...............
and is denoted by r. an 1 = an 2r,
an = an 1r,
Each of the following series forms a geometric
progression Multiplying these n 1 equalities termwise, we
3, 6, 12, 24,..... obtain.
1, 1,  1 (a2a3a4 .... an 1)an = (a2a3 .... an 2 an 1) arn 1.
1, ,.....
3 9 27 After reduction we have an = arn 1.
p, pq2, pq4, pq6,.....
If n be the number of terms, and if " denotes the
The constant factor is also called the common ratio, last, or the nth term, we have " = arn 1
and it is found by dividing any term by that which
immediately preceedes it. In the first of the above This formula enables us to find any term of a
examples the common ratio is 2, in the second it is geometric progression if its first term and common
ratio are known. Therefore it is called the formula
1
; in the third it is q2. of the general term of a geometric progression.
3
Thus, a geometric progression (an) is defined by the Did you notice that the expo-
following conditions : nent on r is 1 less than the sub-
(i) a1 = a where a is called the first term (a z 0); script of a denoting the term number?
(ii) an + 1 = anr for any n t 1
Thus, the formula for the nth term is
If a is the first term and r the common ratio, then n1
an = ar
G.P. can be written as
a, ar, ar2, ar3, ar4,......., arn 1,......... 1 less than the
If, for instance, a = 1 and r = 2, then we have a subscript of a
geometric progression 1, 2, 4, 8, ...... Let us now derive the formula for the VXPRIQWHUPV
RIDJHRPHWULFSURJUHVVLRQ.
1
The conditions a = 4 and r = define the
2 We denote the sum of n terms of a geometric
progression by Sn .
1
geometric progression 4, 2, 1, , .... Let a be the first term, r the common ratio, n the
2 number of terms, and s the sum required.
Note that all the definitions and formulas in the
theory of progressions (both arithmetic and Then Sn = a + ar + ar2 + .... + arn 2 + arn 1;
geometric) remain valid when the terms of the If the common ratio of the progression r is equal to
progressions are complex numbers, but in most 1, then Sn = na1.
problems involving progressions it is assumed (if But if r z 1 , then we proceed as follows: we
not otherwise stated) t hat the terms of the multiply the above equality termwise by r:
progressions are real numbers) rSn = ar + ar2 +.... + arn 2 + arn 1 + arn.
Let (an) be a geometric progression, a its first term, Hence by subtraction,
and r the ratio of the progression. Let us derive the rSn Sn = arn a;
formula for the nth term of a geometric progression. ? (r 1) Sn = a(rn 1);
By the definition of the geometric progression,
a( r n  1)
a2 = ar, ? Sn = ...(1)
r 1
Sequence and Series 

Changing the signs in numerator and denominator, 6ROXWLRQ The first term, a1 is 4. Find r by choosing
a(1  r ) n any term except the first and dividing it by the
Sn = ...(2) 36
1 r preceeding term. For example, r = =3
It will be found convenient to remember both forms 12
given above for Sn, using (2) in all cases except when Since a4 = 108, a5 = 3. 108 = 324. The fifth term also
r is positive and greater than 1. c ould be found by using the formula for
Since arn 1 = ", the formula (1) may be written an.an= a1rn 1, and replacing n with 4, r with 3, and a1
r"  a with 4.
Sn = ; a form which is sometimes useful. a5 = 4 . (3)5 1 = 4 . 34 = 324
r 1
by the formula, an = 4. 3n 1 .
([DPSOH If a, b, c, d be in G.P., show that
(a2 + b2 + c2) (b2 + c2 + d2) = (ab + bc + cd)2
L If a , b , c are in G.P. b2 = a c 6ROXWLRQ Let r be the common ratio of the given

" Q  G.P., then
LL The common difference of a G.P. = b = ar, c = ar2 and d = ar3
D
Now L.H.S. = (a2 + b2 + c2) (b2 + c2 + d2)
where a = first term of G.P., " = last term of G.P.. = (a2 + a2r2 + a2r4)(a2r2 + a2r4 + a2r6)
and n = number of terms of G.P. = (1 + r2 + r4) . a2r2 (1 + r2 + r4)
The common ratio can be zero, positive or = a4r2 (1 + r2 + r4)2 = {a2r(1 + r2 + r4) }2
negative. = (a2r + a2r3 + a2r5)2
a = (a. ar + ar. ar2 + ar2 . ar3)2 = (ab + bc + cd)2
LLL Three numbers in G.P. can be taken as , a, ar;
r
([DPSOH  Find all sequences which are
a ,a
five numbers in G.P. can be taken as 2 , a, simultaneously, an arithmetic and a geometric
r r
progression.
ar, ar2. In general (2m + 1) numbers in G.P. can 6ROXWLRQ Let the numbers a1, a2, .... an, ... form
be written as (m N) an arithmetic progression. Then
a , a ,...., a a n  a n2
, a, ar, ..., arm 1, arm an + 1 = (n t 1).
r m r m 1 r 2
a ,a Since (an) is a geometric progression, applying the
LY Four numbers in G.P. can be taken as , ar,,
r3 r
3
ar ; six numbers in G.P. can be taken as formula of the general form a n = a 1rn 1, where
a1 z 0, we obtain
a , a ,a
, ar, ar3, ar5. a1 r n 1  a1 r n 1
r5 r3 r
a 1r n =
In general : (2m) numbers in G.P. can be written 2
as (m N) or, on reducing by arn 1 z 0
a , 2 ma 3 ,..., a3 , a , ar, ar3, .., ar2m 3, ar2m 1 1  r2
2 m 1 r= ,
r r r r 2
 ([DPSOH  Find a5 and an for the following i.e. (r 1)2 = 0. Hence, r = 1 and the given sequence
geometric sequence is a sequence of equal numbers a1, a1, ..a1,.
Is the sequence 1, 1, 1, .....,1,.... an arithmetic or a
4, 12, 36, 108,....
geometric progression? This sequence can actually
 Comprehensive Algebra

be regarded as an arithmetic progression (with a9 a  15


common difference 0) or as a geometric progression ? = a2 18a + 81 = a2 15a
a a9
(with common ratio 1). 3a = 81 a = 27
([DPSOH Show that 10, 11, 12 cannot be terms Hence A.P. is 27, 18, 9, 0
G.P. is 27, 18, 12, 8
of a G.P.
The sum of the first four terms of A.P. = 54
6ROXWLRQ If possible, let 10, 11 and 12 be the pth,
The second term of G.P. = 18
qth and kth terms of a G.P. whose first term is a and
common ratio is r. ([DPSOH Three positive numbers form a G.P.
If the second term is increased by 8, the resulting
Now 10 = arp 1 ...(1)
sequence is an A.P. In turn, if we increase the last
11 = arq 1 ...(2)
term of this A.P. by 64, we get a G.P. Find the three
12 = ar k 1 ...(3) numbers .
11 q p 6ROXWLRQ Let the numbers be a, a r, a r2
? =r ...(4)
10 where r > 0
12 Hence a, (a r + 8), a r2 in A.P. ...(1)
and = rk q ...(5)
11 Also a, (a r + 8), a r2 + 64 in G.P. ...(2)
11
k q
(a r + 8)2 = a (a r2 + 64)
From (4), = r(q p) (k q) ...(6)
10 4
q p a=  (3)
12 4r
From (5), = r(k q) (q p) ...(7) Also (1) 2 (a r + 8) = (a + a r2)
11
k q q p 16
11 12 (1  r)2 =  (4)
From (6) and (7), = a
10 11
From (3) and (4) r = 3 or  5 (rejected) .
? (11) k q + q p = (10) k q (12)q p Hence a = 4
11k p = 5k q . 2k q . 22 (q p) . 3q p The numbers are : 4, 12, 36
11k p = 5k q . 2k + q 2p . 3q p
([DPSOH Three numbers are successive terms
This is possible only when k p = 0, k q = 0,
of a geometric progression. If we subtract 4 from the
k + q 2p = 0 and q p = 0 or k = p = q which is not third number, these numbers will become successive
possible as p, q, k are distinct. terms of an arithmetic progression. Now if we
([DPSOH If the third and fourth terms of an substract unity from the second and from the third
arithmetic sequence are increased by 3 and 8 term of the resulting arithmetic progression, the
respectively, then the first four terms form a geometric numbers obtained will again become successive
sequence. Find terms of a geometric progression. Find the numbers.
(i) the sum of the first four terms of A.P. 6ROXWLRQ Let the required numbers be a, b, c.
(ii) the second term of the G.P. Now we have b2 = ac
6ROXWLRQ a, (a + d), (a + 2d), (a + 3d) in A.P. Also, 2b = a + c 4.
a, a + d, (a + 2d + 3), (a + 3d + 8) are in G.P. And finally, (b 1)2 = a(c 5)
hence a + d = ar We subtract the third equation from the first. We
obtain a linear equation 2b 1 = 5a relating b and a.
a  d a  2d  3 a  3d  8
also r = = = Expressing now the unknowns a and c from the system
a ad a  2d  3 of linear equations
d3 d5 2b 1 = 5a,
? = d2 + 6d + 9 = d2 + 5d
d d 3 2b = a + c 4
d=9
Sequence and Series 

2b  1 8b  21 ([DPSOH Find a three digit number such that


in terms of b, we get a = ,c= its digits are in G.P., and the digits of the number
5 5
We exclude the unknowns a and c from the system obtained from it by subtracting 400 form an A.P.
substituting their expressions in terms of b into the 6ROXWLRQ Let the digits at hundreds, tens and units
first equation of the system. Then we get a quadratic places be a, ar and ar2 and the required number be x,
equation for b : 9b2 34 b + 21 = 0, whose roots are then
equal to 3 and 7/9. Substituting these values of b x = 100 a + 10 ar + ar2
into the expression for a and c, we get the required Let y = x 400, then
numbers. y = 100(a 4) + 10 ar + ar2 ...(1)
In the number y, the digit at hundreds place is a 4
The numbers are 1, 3, 9 or 1/9, 7/9, 49/4.
Clearly 1 d a 4 d 5 [ 1 d a d 9 and a 4 t 1]
6
5d ad9 ...(2)
([DPSOH Find 6 2.3i According to question a 4, ar, ar2 are A.P.
i 1
? 2ar = a 4 + ar2 or a (r 1)2 = 4
6ROXWLRQ This sum is the sum of the first six terms
of a geometric sequence having a1 = 2.31 = 6 and r1= 2 ...(3)
a
r = 3.
 ar and a are integers.
From the formula for Sn, ? r is a rational number.
? From (2), a must be a perfect square
6 6(1  36 ) 6(1  729)
6 2.3i = S6 = = But from (2), 5 d a d 9 ? a=9
i 1 1 3 2
2 5 1
6( 728) ? From (3), r = 1 = ,
= = 2184. 3 3 3
2 5 1
But r z otherwise ar = 15 ? r =
([DPSOH The sum of the first four terms of a 3 3
Hence a = 9, ar = 3, ar2 = 1
geometric progression is equal to 30 and the sum of Thus required number is 931.
the next four terms is equal to 480. Find the first term
of the progression. ([DPSOH Find the sum
2 2 2
Assume that a is the first term and r is the ratio of the x  1 2 1 n 1
geometric progression. + x  2 + ... + x  n
x x x
1  r1 6ROXWLRQ The required sum can be rewritten in the
a = 30. ...(1)
1 r following way
The next four terms ar4, ar5, ar6, ar7, form a geometric 1 1 1
x2 + x4 + ... + x2n + + 4 + ... + 2 n + 2n.
progression with the initial term ar4 and the ratio r. x2 x x
Therefore, we obtain Summing each of the geometric progressions
4
ar 1  r = 480
4
...(2)
separately and joining the partial sums thus obtained,
1 r we have
2 2
From these equations r4 = 16, which means that x  1
2
2 1 n 1
r = 2 or r = 2. + x  2 + .... + x  n
x x x
If r = 2, then from (1) we find a = 2, and if r = 2, we
get a = 6. ( x 2 n  2  1)(x 2 n  1)
= + 2n.
(x 2  1)x 2 n
 Comprehensive Algebra

([DPSOH Find the sum of n terms of the series ([DPSOH If x1, x2,......xn are n non-zero real
2 + 22 + 222 + ...... numbers such that
6ROXWLRQ Sn = 2 + 22 + 222 + ..... to n terms
( x12  x 22  .....  x 2n 1 ) ( x 22  x 32  .....  x 2n )
2
= [9 + 99 + 999 + ..... to n terms] d (x1 x2 + x2 x3 + .... + xn 1xn)2
9 then prove that x1, x2,..... xn are in G.P.
2 6ROXWLRQ We shall make use of the /DJUDQJHV
= [(101) + (1021) + (1031) + ... + (10n1)]
9 ,GHQWLW\
2
= [(10 + 102 + 103 + ..... + 10n) n] (a 12  a 22  .....  a 2m ) ( b12  b 22  .....  b 2m )
9
(a 1b1 + a 2b2 + .... +a m bm )2
2 10n - 1 = (a1b2a2b1) + (a1b3a3b1)2 + ...
2
= 9 10 . 10 - 1 - n + (am 1 bmam bm 1)2

Thus,
2 10 n
= (10 - 1) - n
9 9 ( x12  x 22  .....  x 2n 1 )(x 22  x 23  .....  x 2n )
(x1 x2 + x2 x3 + .... + xn 1 xn)2 d 0
([DPSOH Find the sum to n terms of the series (x1x3 x2x2)2 + (x1x4 x3x3)2 + .....+
5  9  17  33  .......... (xn 2 xn xn 1 xn 1)2 d 0
2 4 8 16 As x1, x2,.. , xn are real, this is possible if and only if
6ROXWLRQ We have
x1x3 x 22 = x2 x4 x 32 = ...= xn 2 xn x2n 1 = 0
1 1 1
Sn = 2 + + 2 + + 2 +
2 4 8 x1 x2 x3 xn
= = = .... = .
1 x2 x3 x4 x n 1
+ 2 + + ..........n terms
16 Hence x1, x2,..... xn are in G.P.
1 1 ([DPSOH In a certain test, there are n questions,
1 2 n = 2n + 1 1 .
= 2n + In this test 2n i students give wrong answers to atleast
2 1 1 2n i questions (1 d i d n). If the total number of wrong
2
answers given is 2047, find the value of n.
([DPSOH Let Sn be the sum of the first n terms 6ROXWLRQ Number of students giving wrong
of a geometric progression. Prove that answers to atleast i questions = 2n i
Sn (S3n S2n) = (S2n Sn)2. Numbers of students giving wrong answers to atleast
6ROXWLRQ Let the geometric progression be (i + l) questions = 2n i 1.
u1, u2 ...., un, un + 1, ... , u2n, u2n + 1, ..., u3n ? Number of students giving wrong answers to
Hence S3n S2n = u2n + 1 + ......... + u3n, exactly i questions = 2n i 2n i 1
S2n Sn = un + 1 + ..... + u2n.
Also the number of students giving wrong answers
But uk = u1qk 1, us = u1qs 1.
to exactly n questions = 2n n = 20 = 1
Therefore uk = us . qk s , u2n + k = uk q2n,
Consequently, ? Total number of wrong answers
S3n S2n = u2n +1 + ... u3n = q2n (u1 + u2 + ... un) = q2n Sn. 1(2n 1 2n 2) + 2(2n 2 2 n 3) + ..... + (n 1)
S2n Sn = un + 1 + ... u2n = qn (u1 + u2 + ... un) = qn Sn. (21 20) + n (20)
Therefore = 2n 1 + (2n 2 + 2.2n 2) + (2.2n3 + 3.2n 3) +
Sn(S3n S2n) = q2n S 2n , (S2n Sn)2 = q2n S 2n ... + (n 1) 20} + n.20}
Sequence and Series 

= 2n 1 + 2 n 2 + 2n 3 + .... + 20 = 2n 1  Does there exist a geometric progression having


According to the question 20, 21 and 22 as three of its terms?
2n 1 = 2047 2n = 2048 = 211 n = 11
 Find four numbers forming a geometric
progression in which the third term is greater
Practice Problems than the first by 9, and the second term is greater
than the fourth by 18.
 If a, b, c, d are in GP then prove that
(i) (b c)2 + (c a)2 + (d b)2 = (a d)2  Determine the number of the terms of a
(ii) a + b, b + c, c + d are in GP. geometric progression (an) if a1 = 3, an = 96 and
Sn = 189.
 Given four numbers, the first three of which are
three successive terms of a geometri c  The sum of t hree numbe rs which are
progression and the last three are successive consecutive terms of an arithmetic progression
terms of an arithmetic progression. The sum of is equal to 21. Reducing the second number
the extreme numbers is 32 and that of the by 1 and increasing the third number by 1, we
middle numbers is 24. Find the numbers. obtain three consecutive terms of a geometric
progression. Find these numbers.
 Find the ordinal number of the term of the
geometric progression 1280, 640, .., which
3.6 INFINITE GEOMETRIC SERIES
equals 20.
Consider the infinite geometric series
 Prove that if three terms a, b, c form a geometric
1 + x + x2 + x3 + ............upto infinity
progression, then (a + b + c) (a b + c) = a2 +
where x is the common ratio.
b2 + c2.
The series is convergent when |x| < 1 ; the series
 Among the 11 terms of an arithmetic diverges to f when x t 1; the series oscillates
progression, the first, the fifth and the eleventh finitely when x = 1 and oscillates infinitely when
term are three successive terms of a certain x < 1.
geometric progression. Find the arithmetic
3URRI
progression if its first term is equal to 24.
The sum of the first n terms
 Find the sum of the following series:
1  xn
1 1 1 Sn ; x z 1.
1 + ..... to n terms. 1x
2 4 8
 The sequence {bn} is a G.P. with L If |x| < 1, xn o 0 as n o f.
b4 1 1 as n o f .
? Sn
b 6 = 4 and b2 + b5 = 216. 1 x
108 Hence when |x| < 1, 6xn1 is convergent and
Show that b1 = 12 or
7
1
 Find the least value of n such that its sum is .
1 x
1 + 2 + 22 + .... + 2n 1 t 300.
LL If x > 1, xn o f as n of.
 The sum of n terms of a series is a. 2n b, find its ? Sn of as n of.
nth term. Are the terms of this series in G.P. ?
 Comprehensive Algebra

If x = 1, Sn = n of as n o f. f n 1
D

Therefore, 6xn1 is divergent and diverges to f. Sf = 6 3 1 = a
n 1 2  U
LLL If x = 1,
(the first term) = 3 and r (the common ratio)
Sn = 0 or 1 according as n is even or odd.
1
? Sn o0 or 1 according as n is even or odd. = = 3 =6
Moreover |Sn| d 1. 2 1 1
Hence (Sn) oscillates finitely. 2
Therefore, 6xn1 oscillates finitely. Thus, the sum of the given infinite geometric series
is 6. Putting in an informal way, as we continue
LY If x < 1, 1 xn of or f according as n is
a dding more and more terms, the sum is
odd or even.
approximately 6.
Thus Sn of or f according as n is odd or even. 3  3  3  3  ......
Hence (Sn) oscillates infinitely. We can visualize 3 + =6
2 22 2 3 2 4
Therefore, the series 6xn1 oscillates infinitely. by writing the formula for the sum as a function of x,
Thus, WKHVXPRIDQLQILQLWHJHRPHWULFVHULHV namely
S = a + ar + ar2 + ......... upto infinity, when D   U [ D DU [
f(x) = = 
|r| < 1 is  U  U  U
S=
a
1
1 r With a = 3 and r = , the sum function is
1 1 1 2
Consider the series 1, , 2 , 3 ,...
2 2 2 x
3 1
1  1n 3  2 x
2 f(x) = = 6 6 1
The sum to n terms = 1 1 1 1 2
1 1 2 2
2
Y y=6
1 1
= 2 1  n 2  n 1 .
2 2
x
From this result it appears that however many terms f(x) = 6 6 1
2
be taken the sum of the above series is always less
than 2.
0 X
Also we see that, by making n sufficiently large, we Figure shows the graph of f(x) and y = 6. Since the
1 graph of Sn consists of points on the graph of f, we
can make the fraction as small as we please. see that Sn o 6 as n o f.
2 n 1
Thus by taking a sufficient number of terms the sum There are some infinite geometric series to which we
can be made to differ by as little as we please from 2. cannot assign a sum, such as
([DPSOH Find the sum of the infinite series : 2 + 4 + 8 + 16 + 32 + 64 + .... .

f n 1 As more and more terms are added, the sum is getting


3 3 3 3
6 3 1 = 3 +  2  3  4  ...... larger and larger without bound.
n 1 2 2 2 2 2
6ROXWLRQ We can find the sum by using the  ([DPSOH  Find the sum of the infinitely
formula for the sum of an infinite geometric series. decreasing geometric progression the sum of whose
Sequence and Series 

first three terms is equal to 7 and the product of those 92 9 u 3 27


Sf = = =
terms is equal to 8. 1  ( 2 3) 2 2
Assume that a is the first term and r is the ratio of the ([DPSOH If x = 1 + a + a2 + a3 + ....fand
geometric progression. y = 1 + b + b2 + b3 + ..... fshow that

D   U  U   xy
From the question, 1 + ab + a2b2 + a3b3 + .... f= where
x  y 1
D U  
From the second equation of the system we find 0 < a < 1 and 0 < b < 1
ar = 2 and substituting a = 2/1 into the first equation 6ROXWLRQ Given x = 1 + a + a2 + a3 + ... f
of the system, we obtain the quadratic equation 1
2r2 5r + 2 = 0 whose roots are r = 1/2 and r = 2. The ? x=
1a
seque nce is infinitely decreasing, |r| < 1. x ax = 1
Consequently, r = 1/2, and in that case a = 4. The sum x 1
a= ...(1)
of that progression can be found by the formula: x
and y = 1 + b + b2 + b3 + .... f
D
S= = 8. y 1
 U
Similarly b = ...(2)
([DPSOH If the first 3 consecutive terms of a y
geometrical progression are the roots of the equation since 0 < a < 1, 0 < b < 1
2x3 19x2 + 57x 54 = 0 find the sum to infinite ? 0 < ab < 1
number of terms of G.P. Now 1 + ab + a2 b2 + a3b3 + ....f
6ROXWLRQ 2x3 19x2 + 57x 54 = 0 1
=
1  ab
D 1
let the roots be , a, ar = {from (1) & (2)}
U x  1 y 1
1
19 x y
? a 1  r  1 = ...(1)
r 2 xy
xy
= =
57 xy  xy  x  y  1 x  y  1
a2 1  r  1 = ...(2)
r 2  ([DPSOH  Find the sum of an infinitely
54 = 27 decreasing G.P. whose first term is equal to (b + 2)
a3 1  r  1 = ...(3) and the common ratio is 2c1, if b is the least value
r 2
of the product of the roots of the equation (m2 + 1)x2
? a=3 3x + (m2 + 1)2 = 0 and c is the greatest value of the
19r sum of its roots.
from (1) 3(r2 + r + 1) =
2 6ROXWLRQ b = least value of product of the roots of
6r + 6r + 6 = 19r 6r2 13r + 6 = 0
2 the given equation
2 3
(2r 3)(3r 2) = 0  r = or (rejected) (m 2  1) 2
3 2 = least value of = least value of
m2  1
D 9
? Numbers are , a, ar , 3, 2 m2 + 1, which is 1
U 2
 Comprehensive Algebra

c = greatest value of sum of the roots 3 5

3 1  1 1  1
= greatest value of , which is 3
2
2  2
m 1 6ROXWLRQ Sn = 1 + 1 .......
? the first term of the G.P. = 3 and the common 1 1 1
2 2
2 n terms
ratio is 3 5
3
= 1 + 2 1  1  2 1  1 + .........
a 3
? Sf = = 9 2 2
1 r 1 2 n times
3 = 1 + (2 + 2 + ......... (n 1) times)
([DPSOH If Sn and S denote the sum to n terms 2 4

1 1 1  1  .........(n  1)times
and to infinity of the series 2 + 1 + 1 + ....., then 2 2
2 8
(1/2)2 1
n 1
1 = 1 + 2 (n 1)
find the value of n such that S Sn < 2
1
1000 1 2
1
6ROXWLRQ Here a = 2, r =
3 2
4
a 1 2 4 1  1
? S= =
2
=8 = 2n 1 + 2 n = 2n
1 r 1 3 3 3 .2 3 22n
4

Sn =
>
a(1  r ) 2 1  (3 / 4)
n
=
n
3
= 8 8
n
@ 3URSHUWLHVRI*3
1 r 1 3 4 A geometric progression (an) possesses the following
4 characteristic property: the square of any of its
n
3 1 terms, beginning with the second, is equal to the
? S Sn = 8 < product of its predecessor and successor:
4 1000
if (0.75)n < 0.000125
a 2n 1 = anan + 2 (n t 1).
if n log100.75 < log100.000125
if n (1 .8751) < 4 .0969 3URRI

4.0969 By the definition of the geometric progression,


if n> = 31.25 a n 1 a n  2
1 .8751
? n = 32 an a n 1 = r a 2n 1 = anan + 2
f 2 n11 The converse is also true ; if a certain sequence (an)
([DPSOH  Let P = 10 then find

n 1 is such that a 2n 1 = anan + 2 and a1 z 0, then this
log0.01(P). sequence (an) relationship be fulfilled. Then
1 1 1
1 2 3
6ROXWLRQ P = 10 10 2 10 2 10 2 ............ a n 1 a n2
1 an a n 1
1 1  12  13 ......  f 1 1
2 2
P = 10 2 = 10 2 = 102 = 100 that is, the ratio of any term of the sequence (an) to
? log0.01(P) = 1 its predecessor is equal to one and the same number.
Hence, (an) is a geometric progression.
([DPSOH Find the sum to n terms of the series,
1 1 1 1 1 1 Thus, the established property is inherent in a
1 + 1 + + 2 + 1 + + 2 + 3 + 4 + ...... geometric progression and only in it.
2 2 2 2 2 2
Sequence and Series 

In the case of a geometric progression with positive YLL If a1, a2, a3,...................., are in G.P.
terms, the above relationship may be written in the
then ar = a r  k a r  k  k, 0 d k d n r
form an + 1 = a na n2 .
([DPSOH Let r be the common ratio of the GP
A geometric progression with positive terms a1, a2, a3, ...., an. Show that
possesses the following characteristic property : any 1 1 1
of its terms, beginning with the second, is a + + ...... +
a1m + am
2 am
2 + am
3 am
n -1 + am
n
geometric mean of its predecessor and successor.
,PSRUWDQW3URSHUWLHVRI*3 1 - r m(1- n)
=
a1m (r m - r -m )
L If a1, a2, a3, ........................ are in G.P. then
(a) a1k , a2k, a3k .............. are in G.P. 6ROXWLRQ LHS
a1 a 2 a 3 1 1 1
(b) , , .............. are in G.P.. = a m  (a r ) m  a m  (a r ) m  ..... a m  (a r ) m
k k k 1 1 2 2 n 1 n 1

If each term of a GP be multiplied or divided


= 1 1  1  .....  1
by the same non-zero quantity, the resulting
1  r m a 1m a m2 a mn 1
sequence is also a GP .
As a1 , a2, a3,.... form a GP, their reciprocals
(c) a1 r k , a2 r k, a3 r k .... are not in G.P.,
k z
1 , 1 , 1 ,....
a1 a 2 a 3
(d) a 1r , a 2r , a 3r , ...., a rn are in G.P..
1
are also in GP whose common ratio = .
(e) 1 , 1 , 1 ................... are in G.P.. r
a1 a 2 a 3 1
m

m

m

Again, their powers , 1 , 1 , .... are


LL If a1, a2, a3, ........ and b1, b2, b3, ........are in G.P. a1 a2 a3
then a1b1, a2b2, a3b3,................ are in G.P. and m
1
a1 a 2 a 3 also in GP whose common ratio will be .
r
b1 , b 2 , b3 .............. are in G.P.. n 1
1 1  1
m m
LLL If a1, a2, a3, ........ and b1, b2, b3, ........are in G.P. 1 . a1 r
then a1 r b1 , a2 r b2 , a3 r b3 ............ are not in ? LHS =
1  rm 1  1m
G.P. But note that addition and substraction of r
two G.P. when common ratio of both are same, 1  r  mn  m 1  r m (1n )
is G.P. = m m = m m
m
a1 (1  r )(1  r ) a 1 (r  r  m )
LY If a1, a2, a3, ..... are positive numbers in G.P. then
([DPSOH Three positive distinct numbers x,
log a1, log a2, log a3, .... are in A.P. & vice-versa.
y, z are three terms of geometric progression in an
Y If a1, a2, a3, ........................ an are in G.P. order and the numbers x + y, y + z, z + x are three
then a1n1 = a2an 1 = a3 an 2 = ..............
terms of arithmetic progression in that order. Prove
YL If a1, a2, a3, ........................ are in G.P. that
then a 22 = a1a3 ; a 23 = a2a4 ; a 24 = a3 a5 ; ...... xx . yy . zz = xy . yz . zx.
 Comprehensive Algebra

6ROXWLRQ Let x be first term of G.P. and y and z be consisting of as many nines as there are recurring
the mth and nth of same G.P. respectively figures followed by as many zeros as there are non-
recurring figures.
tm = y = xrm 1
and tn = z = x rn 1, where r is a common ratio of G.P. ([DPSOH  Using G.P. express
m  1 log(y / x )
p
0. 3 and 1.2 3 in form.
n  1 log(z / x) ...(1) q
6ROXWLRQ Let x = 0. 3 = 0.3333.............
Now, we have y + z = x + y + (m 1) d
and z + x = x + y + (n 1) d, where d is common = 0.3 + 0.03 + 0.003 + 0.0003 + .........
difference of A.P. 3  3  3  3  .......
=
m 1 z  x 10 100 1000 100000
n 1 z  y 3
= 10  3 1.
log( y / x) zx 1
From (1) and (2), 1 9 3
log(z / x) zy 10
z x Let y = 1 .2 3 = 1.233333 = 1.2 + 0.03 + 0.003
y z z y + 0.0003 + ........
y = z(z x) /(z y) . x(x y)/(z y)
x x 3 3 3
= 1.2 + 2  3  4 + .......
y(z y) = z(z x) . x(x y) 10 10 10
xx . yy . zz = xy . yz . zx 3
10 2 1.2  1 37 .
5DWLRQDO 5HSUHVHQWDWLRQ RI D 5HFXUULQJ = 1.2 + 1
1 30 30
'HFLPDO 10
Let P denote the figures which do not recur, and
suppose them p in number; let Q denote the recurring Practice Problems
period consisting of q figures, let D denote the value
of the recurring decimal; then 11 1  1
 Sum + ...... to f
5 7 52 7 2
D = 0.PQQQ.,,, ;
? 10n D = P.QQQ.... ; 1  1  1 .... to f

and 10p + q D = PQ.QQQ....;  Find the value of 9 3 9 27 .


 The sum of an infinite G.P. whose common ratio
therefore, by subtraction, (10p + q 10p) D = PQ P; is numerically less than 1 is 32 and the sum of
that is, 10p (10q 1) D = PQ P; the first two terms is 24. Find the terms of G.P.
PQ  P  The sum of an infinitely decreasing geometric
? D=
(10 q  1)10 p progression is 16 and the sum of the squares of
Now 10q 1 is a number consisting of q nines; 3
its terms is 153 . Find the fourth term and the
therefore the denominator consists of q nines 5
followed by p zeros. Hence we have the following common ratio of the progression.
rule for reducing a recurring decimal to a fraction :  If S be the sum, P the product & R the sum of
For the numerator subtract the integral number the reciprocals of a G.P., find the value of
consisting of the non-recurring figures from the n
integral number consisting of the non-recurring and 2 R
P .
recurring figures; for the denominator take a number S
Sequence and Series 

 The first term of an infinitely decreasing 1


geometric progression is unity and its sum is S. The general term of an H.P. is a n =
a  ( n  1)d
Find the sum of the squares of the terms of the
In case we have to assume a certain number of terms
progression.
which are in H.P., then suggested choice is as follows:
f n n
1 ,1, 1
 Find the sum : 6 1  3 L for three terms use
n 3 4
4 a d a a d
 Between what numbers must x lie in order that 1 , 1 , 1 , 1
LL for four terms use
a  3d a  d a  d a  2d
2 3
2 x  2x  2 x  .....
the series
x 3 x 3 x 3

to f may have a finite sum ?  There is no general formula for finding out the
sum of n terms of H.P.
 Prove that in an infinite GP, whose common
ratio r has its absolute value less than 1, the  No term of H.P. can be zero.
 For HP whose first term is a and second term
ratio of any term to the sum of all the succeeding
ab
1 r is b, the nth term is t n =
b(n1)(ab) .
terms is .
r
2  1  1  1  .............
2ac a a b
 Sum  If a, b, c are in HP  b = or = .
2 1 2  2 2 a c c bc
 Questions in H.P. are generally solved by
 Solve the equations : inverting the terms, and making use of the
(i) 1 + a + a2 + .... + ax = (1 + a) (1+ a2) properties of the corresponding AP.
(1 + a4)(1 + a8)
5HFRJQL]DWLRQRI$3*3+3
(ii) 2x + 1+ x2 x3 + x4 x5 + ... = 13/6 if |x| < 1
loga x Let a, b, c be three successive terms of a sequence.
(iii) (3(1 1/2 + 1/4 1/8 + ...))
ab a
= (20(1 1/4 + 1/16 1/64 + ....)) logx a L If , then a, b, c are in A.P..
b c a
(iv) 2x 1 + 2x 4 + 2x 2 = 6.5 + 3.25 + 1.625 + ... ab a
LL If , then a,b, c are in G.P..
b c b
 The continued product of three numbers in G.P.
ab a
is 216 and the sum of the products of them in LLL If , then a, b, c are in H.P..
b c c
pairs is 156; find the numbers.
25

3.7 HARMONIC PROGRESSION (HP) ac


L a, b,c are in A.P. if and only if b =
2
A sequence is said to HP if the reciprocals of its LL a, b, c are in G.P. if and only if b2 = ac
terms are in AP . 2ac
LLL a, b, c are in H.P. if and only if b =
If the sequence a 1, a 2, a 3, .... , a n is an HP, then ac
In general we can use the following :
1/a 1, 1/a 2, .... , 1/a n is an AP. If tn tn 1 = constant for all n t 2 then the sequence
is in AP.
 Comprehensive Algebra

tn 2ac
If = constant, for all n t 2 then the sequence a + c = b or a + b =
t n 1 ac
is in GP. a, b, c are in H.P. or a + c = b.

If 1  1 = constant, for all n t 2 then the  ([DPSOH  If a1, a2, .... , an are in harmonic
t n t n 1 progression, show that
sequence is in HP. a1a2 + a2a3 + ..... an1an = (n 1)a1an.
([DPSOH If a, b, c are in H.P., show that 6ROXWLRQ Here 1 , 1 , ..... , 1 are in A.P..
a , b , c a1 a 2 an
are also in H.P..
bc ca ab Let the common different be d.
6ROXWLRQ Given, a, b, c are in H.P. 1, 1 a1  a 2
? d or a1a 2 ,
1, 1 ,1 a 2 a1 d
are in A.P..
a b c 1  1 d a2  a3
or a2a3
abc abc D EF a3 a 2 d
, , are in A.P.. ..................... .... .....................
a b F a n 1  a n
1  1 d or a n 1a n
bc ca ab a n a n 1 d
1+ ,1+ ,1+ are in A.P..
a b c
By addition,
a b c
, , are in H.P.. 1  1 ( n  1)d
bc ca ab
a n a1
a1  a n
([DPSOH If 1  1  1  1 0 prove and a1a2 + a2a3 + ..... + an1an = .
a c ab cb d
that a, b, c are in HP unless b = a + c. a1  a n a1  a n
Now (n1)d = or = (n 1)a1an.
a 1a n d
6ROXWLRQ
? a1a2 + a2a3 + ..... + an1an = (n 1)a1an.
a  c  c a  a  b
ac (a  b )(c  b = 0 ([DPSOH If a2, b2, c2 are in A.P., show that b + c,
c + a, a + b are in H.P.
a  c  (a  c )  2 b
ac ac  b(a  c )  b 2 = 0 6ROXWLRQ By adding ab + ac + bc to each term, we
see that
O O  2b a2 + ab + ac + bc, b2 + ba + bc + ac, c2 + ca + cb
Let a + c = O ? + =0
ac ac  bO  b 2 + ab are in A.P.,
acO  bO2  b 2 O  acO  2abc that is (a + b) (a + c), (b + c) (b + a), (c + a) (c + b) are
=0
ac(ac  bO  b 2 ) in A.P.,
acO bO2 + b2O + acO 2abc = 0 ? dividing each term by (a + b) (b + c) (c + a),
2ac(O b) bO (O b) = 0 1 , 1 , 1
(2ac bO) (O b) = 0 b  c c  a a  b are in A.P.,
2ac that is, b + c, c + a, a + b are in H.P.
O = b or O =
b
2ac 343
a + c = b or a + c = (  a + c = O) ([DPSOH The value of x y z is 55 or
b 55
according as the series a, x, y, z, b is an A.P. or H.P.
Sequence and Series 

Find the values of a & b given that they are positive  If a, b, c, be in A.P.; p, q, r be in H.P.; and ap, bq,
integers.
6ROXWLRQ Let a, x, y, z, b are in A.P. b = a + 4d p r ac.
cr be in G.P. show that 
r p c a
ba
d=  If a, b, c are in H.P. then prove that
4
a 3b3 + b3c 3 + c 3a3 = (9 ac  6 b2) a 2c 2 .
3a  b ab
x=a+d= y=a+2d= 15
4 2  The value of xyz is if a, x, y, z, b are in AP,,
2
3a  b
& z= 18
4 while xyz is if a, x, y, z, b are in HP. If a and
5
Similarly when a, x, y, z, b are in H.P. then
ab b are positive integers find them.
d=
4 ab

Hence x =
4 ab
; y=
2ab
; z=
4 ab 3.8 ARITHMETIC, GEOMETRIC &
a  3b ab 2a  b HARMONIC MEANS
In the first case
3a  b a  b 3a  b $ULWKPHWLF0HDQ
. . = 55  (1)
4 2 4 When three quantities are in arithmetic progression,
In the second case the middle one is said to be the arithmetic mean of
343 the other two.
4ab 2ab 4 ab
. . =  (2) Thus, a is the arithmetic mean between a d and a + d.
a  3b a  b 2a  b 55
7RILQG WKHDULWKPHWLF PHDQEHWZHHQ WZRJLYHQ
dividing TXDQWLWLHV.
a 3 b3 = 73 a = 7 ; b = 1 or a=1 ; b=7
Let a and b be the two quantities and A the arithmetic
mean.
Practice Problems Since a, A, b are in A.P. we must have
b A = A a,
1 8  4 each being equal to the common difference ;
 Find the 7th term of the series ....
3 23 11 ab
 Find the 4th term of an H.P. whose 7th term is ? A= 2
1 1 The arithmetic mean of a set of any n numbers
and 13th term is .
20 38
a1 a 2 a 3 .....a n
 Find the largest positive term of the H.P. whose a 1, a 2, ... , a n is A = .
n
first two terms are 2/5 and 12/23.
7RLQVHUWDJLYHQQXPEHURIDULWKPHWLFPHDQVEHWZHHQ
 If a, b, c, d be in G.P. and ax = by = cz = dw, prove
WZRJLYHQTXDQWLWLHV
that x,y, z, w are in H.P.
Let a and b be the given quantities, n the number of
 If ", m, n are three numbers in G.P., prove that means.
the first term of an A.P. whose "th, mth, and nth
terms are in H.P. is to the common difference as Let A1, A2, A3,....., An be the A.M.'s between them.
m + 1 to 1. Then a, A1, A2, A3,...., An, b will be in A.P.
 Comprehensive Algebra

Including the extremes the number of terms will be 6ROXWLRQ We have to find three numbers a2, a3,
n + 2; so that we have to find a sequence of n + 2 and a4 such that the sequence 3, a2, a3, a4, 19 is an
terms in A.P., of which a is the first, and b is the last. arithmetic progression.
Let d be the common difference
Let the difference of this progression be equal to d.
 ([DPSOH  Find the first four terms for the By the formula of the general term of an arithmetic
sequences defined as follows : progression, 19 = 3 + 4d or d = 4.
then b = a + (n + 2 1) d Thus we have obtained the progression 3, 7, 11, 15,
b a 19. The required numbers are 7, 11 and 15.
? d = n 1
([DPSOH Insert 20 A.M.s between 2 and 86.
? A1 = a + d, A2 = a + 2d, ............, An = a + nd 6ROXWLRQ Here 2 is the first term and 86 is the 22nd
ba ba term of A.P.
A1 = a + , A2 = a + 2 , .........., so 86 = 2 + (21) d d=4
n 1 n 1
so the sequence is 2, 6, 10, 14,......., 82, 86
ba ? The required means are 6, 10, 14,.....82
An = a + n
n 1
Thus, the n A.M.s between a and b are ([DPSOH Find four numbers in A.P. whose sum
is 6 and the product of whose extremes is 10 times
2( b  a ) n( b  a )
a  ba, a  , ...., a  the product of the means.
n 1 n 1 n 1
6ROXWLRQ Let the four numbers be
For example, since 1, 3, 5, 7, 9, 11 are in A.P., therefore a 3d, a d , a + d, a + 3d.
3, 5, 7, 9 are the four A.M.s between 1 and 11. 3
Sum of these numbers = 4a = 6 ?a=
2
The sum of all the n A.M.s inserted Product of extremes = 10 times product of means
between a and b is equal to n times (a 3d)(a + 3d) = 10 (a d) (a + d)
the single A.M. between a and b 9
? 10(a2 d2) = a2 9d2 or d2 = 9a2 = 9
n 4
i.e. Ar = A1+ A2+ .....+ An = nA 9
r 1 Hence d =
2
where A is the single A.M. between a and b. 9
Let two numbers a and b, and A1, A2, A3,...., An are When d = , we get the four numbers as 12, 3, 6, 15
2
A.M.'s between them 9
If d is the common difference of thisA.P. then A1 = a + d= gives the same numbers in reverse order..
2
d, An = second term from end = b d ([DPSOH Between two numbers whose sum is
n
? A1 + A2 + A3 + .... +An = (first term + last term) 13
2 , an even number of A.M. s is inserted, the sum of
6
n n
= (A1 + An) = (a + d + b d) these means exceeds their number by unity. Find the
2 2
number of means.
a b 6ROXWLRQ Let a and b be two numbers and 2n
= n = n (A.M. of a and b)
2 A.M. s are inserted between a and b then
([DPSOH Insert three arithmetic means between 2n
the numbers 3 and 19. (a + b) = 2n + 1.
2
Sequence and Series 

13 13 b2
n = 2n + 1. given a  b n=6 = a 2 + a b (n + 1) + (2 n2 + 3 n + 1)
6 6 6
? The number of means = 12. 2
(a  b)  (a  2 b)  .......  (a  n b)
([DPSOH  Two consecutive numbers from Again
n
1, 2, 3,...., n are removed. The arithmetic mean of
2
105 n 1
the remaining numbers is . Find n and those = a  b
4 2
removed numbers. b2 2
6ROXWLRQ Let p and (p + 1) be the removed = a 2 + a b (n + 1) +(n + 2 n + 1)
numbers from 1, 2,...,n then sum of the remaining 4
Hence A.M. of squares  square of A.M.
n (n  1)
numbers = (2p + 1) n 2  1
2 = b2
From given condition 12
n(n  1)
 (2p  1) *HRPHWULF0HDQ
105 2
4 n2 When three positive quantities are in geometric
2n2 103n 8p + 206 = 0 progression, the middle one is said to be the
Since n and p are integers so n must be even let n = 2r geometric mean of the other two. Thus, if a, b, c are
in GP, b is the G.M. between a and c ( a , b, c > 0).
4 r 2  103(1  r )
we get p = 7R ILQG WKH JHRPHWULF PHDQ EHWZHHQ WZR JLYHQ
4
TXDQWLWLHV.
Since p is an integer then (1 r) must be divisible by 4.
Let a and b be the two positive quantities and G the
Let r = 1 + 4t, we get
geometric mean.
n = 2 + 8t and p = 16t 2 95t + 1,
Now 1 d p < n Since a, G, b are in G.P., we must have
1 d 16t 2 95t + 1 < 8t + 2 t=6 G2 = ab
n = 50 and p =7 ? G = ab
Hence removed numbers are 7 and 8. 7KH JHRPHWULFPHDQRI D VHW RI DQ\ Q SRVLWLYH
([DPSOH Prove that the arithmetic mean of QXPEHUVDDDQLV
the squares of n quantities in A.P. exceeds the G = (a1a2 ... an)1/n
squares of their arithmetic mean by a quantity which
7RLQVHUWDJLYHQQXPEHURIJHRPHWULFPHDQVEHWZHHQ
depends only upon n and upon their common WZRJLYHQSRVLWLYHTXDQWLWLHV
difference .
6ROXWLRQ a + b , a + 2 b , ........ , a + n b Let a and b be the given positive quantities, n the
number of means.
(a  b )2  (a  2 b ) 2  .......  (a  n b) 2
Hence Let G1, G2, G3 , ...., Gn be the n G.M.s between a and b.
n
Then a, G1, G2, G3,...., Gn, b will be in G.P.
= 1 n a 2 + nab (n + 1) + b 2
n (n + 1) (2 n + 1)
n 6 In all there will be n + 2 terms; so that we have to find
b 2 a series of n + 2 terms in G.P., of which a is the first
= a 2 + a b (n + 1) + (n + 1) (2 n + 1) and b the last.
6
 Comprehensive Algebra

Let r be the common ratio; ([DPSOH Insert three geometric means between
then b = the (n + 2)th term = arn + 2 1= arn + 1 the numbers 1 and 256.
b 6ROXWLRQ We have to find three numbers a2, a3
rn + 1 = and a4 such that
a
1
the sequence 1, a2, a3, a4, 256 becomes a geometric
n 1 progression.
? r= b .
a Let the ratio of this progression be equal to r.
? G1 = ar, G2 = ar2 ,...., Gn = arn By the formula of the general term of a geometric
progression, 256 = 1.r4, or 44 = r4, or r = 4. We have
l / n 1 2 / n 1
b b obtained the progression 1, 4, 16, 64, 256. The
G1 = a , G2 = a ,..., required numbers are 4, 16 and 64.
a a
n / n 1 ([DPSOH The sum of four numbers in G.P. is 60
b and the arithmetic mean of the first and last numbers
Gn = a
a is 18. Find the numbers.
)RUH[DPSOH 6ROXWLRQ Let the four numbers be a, ar, ar2, ar3
Since 1, 2, 4, 8, 16 are in G.P., therefore 2, 4, 8 are the a(1 + r + r2 + r3) = 60 ...(1)
three G.M.s between 1 and 16. a  ar 3
= 18
2
The product of all the n G.M.s between ? a(1 + r)(1 + r2) = 60 and a(1 + r)(1 r + r2) = 36
a and b is equal to the nth power of 1  r2 5
Division gives =
the single GM between a and b. 1  r  r2 3
n
i.e. Gr = (G)n where G is the single GM 1
? 2r2 5r + 2 = 0 giving r = 2 or
r 1 2
between a and b. Putting r = 2 in equation (1), we get a = 4
Let two numbers a and b, and G1, G2, G3,..., Gn are ? The numbers are 4, 8, 16, 32
A.M.'s between them 1
r= gives the same four numbers.
2
Then a, G1, G2, G3,... Gn, b are in G.P. If r is the common
ratio then +DUPRQLF0HDQ
G1 = ar, G2 = ar2, G3 = ar3 ,.....,
b = 2ac/[a + c] .
b When three quantities are in harmonic progression,
Gn = 2nd term from last =
r the middle one is said to be the harmonic mean of the
b other two. If a, b, c are in HP, b is the H.M. between a
Gn - 1 = 3rd term from last = and c.
r2
? G1G2G3 .... Gn = ar.ar2...arn 1 = an r1 +2 +........(n1) 7R ILQG WKH KDUPRQLF PHDQ EHWZHHQ WZR JLYHQ
= anrn(n1)/2 TXDQWLWLHV.
= (a2rn1)n/2 Let a and b be the two quantities and H the harmonic
mean.
= ( ab ) n
Since a, H, b are in H.P. we must have
= (G.M. of a and b)n
2 11
H a b
Sequence and Series 

2ab The sum of reciprocals of n Harmonic


? H= a  b means between two given numbers is n
times the reciprocal of single H.M.
7KHKDUPRQLFPHDQRIDVHWRIDQ\QQXPEHUVD between them.
DDQLV
Let two numbers a and b and H1, H2, H3,....., Hn are
1 H.M.'s between them then a, H1, H2, H3, .....Hn, b are

1  1  ...  1 in H.P. If d is the difference of the corresponding A.P.
a1 a 2 an 1 1  d, 1 1  2d, 1 1  3d,....
H= ?
n H1 a H2 a H3 a

1 1
H n = 2nd term from last = b d
or 1 n
H 1  1  .....  1 1 1
a1 a 2 an H n 1 = 3rd term from last = b 2d
7RLQVHUWDJLYHQQXPEHURIKDUPRQLFPHDQVEHWZHHQ 1  1  1  ...  1  1  1
WZRJLYHQTXDQWLWLHV ? H1 H 2 H 3 H n 2 H n 1 H n
Let a and b be two given numbers and H1, H2, H3,....,
Hn are H.M.'s between them. n 1 1 n1 1
= 2 H  H =  d   d
Then a, H1, H2, H3, ....., Hn, b will be in H.P. 1 n 2 a b
If d is the common difference of the corresponding n1  1
A.P. =
2a b
? b = (n + 2)th term of H.P.

= 1 ([DPSOH Insert 4 H.M.s between 2 and 2 .


( n  2)th term of corresponding A.P. 3 13
6ROXWLRQ Let d be the common difference of
1 corresponding A.P.
? b= 1
 ( n  2  1)d
a 13  3
so d= 2 2 =1
1 1 5
+ (n + 1) d
b a
(a  b ) 1 3 1 5 2
d= ? H1 2 2 or H1 =
(n  1)ab 5

1 1 1 1 1 1 1 32 7 2
? = + d, = + 2d,...., = + nd H2 2 2 or H2 =
H1 a H2 a Hn a 7

1 1 (a - b) 1 1 2(a - b) 1 3 3 9 2
= + , = + ,....., H3 2 2 or H3 =
H1 a (n + 1)ab H 2 a (n + 1)ab 9

1 1  n( a  b ) 1 3  4 11 2
H4 2 2 or H4 = .
Hn a (n  1)ab 11
 Comprehensive Algebra

([DPSOH  A, B and C are distinct positive ab


ab  =4
integers, less than or equal to 10. The arithmetic mean (5  ab
of A and B is 9. The geometric mean of A and C is
5 ab + ab ab = 20 + 4 ab
6 2 . Find the harmonic mean of B and C.
6ROXWLRQ A + B = 18 ...(1) ? ab = 20 or ab = 400
AC = 72 ...(2) ab
There are only two possibilities ? = 5 + ab = 25 ? a + b = 50
2
A = 10 and B = 8 or A = 8 and B = 10 Since a + b = 50 and ab = 400
If A = 10 then from (2) C is not an integer. a = 40 and b = 10
Hence A = 8 and B = 10; C = 9
([DPSOH The arithmetic mean of two numbers
2 10 9 180
? H.M. between B and C = = 3
exceed their geometric mean by and the geometric
10  9 19
2
([DPSOH If a be the A.M. and H the H.M. 6
mean exceed their harmonic mean by , find the
between two quantities a and b, prove that 5
numbers.
aA . bA A 6ROXWLRQ Let the two numbers be a and b and A, G,
aH bH H H be the A.M. , G.M. and H.M. respectively between a
ab 2ab and b.
6ROXWLRQ Given, A = and H =
2 ab a+b 2ab
Then, A = ; G = ab and H = ...(1)
2 a+b
aab
a A. b a 2 3 6
Now, = Given, A = G + and G = H +
a  H b  H b  ab
2 2 5
ab 6
or, H=G ...(2)
5
(a  b)(b  a )(a  b) 2
=
? G + G = G2
4(a  ab  2ab)(ab  b 2  2ab)
2 3 6
 AH = G2
2 5
(a  b)(b  a )(a  b)2 (a  b) 2 or 2
(2G + 3) (5G 6) = 10G or 3G = 18
= =
4.a(a  b).b( b  a ) 4ab ? G= 6
3 15
ab From (2), A = 6 + = or a + b = 15 ...(3)
A 2 2
= 2 =
2ab H Also G = 6 ? ab = 6 or ab = 36 ...(4)
ab 2 2 2
Now (a b) = (a + b) 4ab = 15 4.36 = 81
 ([DPSOH  The A.M. between two positive ? a b=9 ...(5)
numbers exceeds the G.M. by 5, and the G.M. exceeds from (3) and (5), a = 12, b = 3 or a = 3, b = 12
the H.M. by 4. Find the numbers. Hence the numbers are 3 and 12.
6ROXWLRQ If the numbers are a and b,
([DPSOH If p be the first of n A.M.s between
a  b  ab = 5 and ab  2ab = 4 two numbers; q be the first of n H.M.s between the
2 ab same numbers, prove that the value of q cannot lie
ab 2
Substituting for in the second equation, n + 1
2 between p and
n 1
p.
we get
Sequence and Series 

6ROXWLRQ Let the two numbers be a and b. Since n 2


A.M.'s have been inserted between a and b n  1
(q p) q  p
ba n  1 t 0
? Common difference of A.P., d = By wavy curve method.
n +1
2
b a an + b n 1
= ? q d p and q t p
n + 1 n + 1
Now p = a + d = a + ...(1) n 1
Since n H.M.s have been inserted between a and b.
Then common difference of the corresponding A.P. 
11 S 2
(a  b) p n  1
D= b a n 1
( n  1) ab( n  1)
1 1 D 1  (a  b ) n 1
2
? q a a ab( n  1) Hence q cannot lie between p and p
n 1
ab(n + 1)
Then q =
(a + bn)
...(2) $OWHUQDWLYH0HWKRG
Since n + 1 > n 1
$OWHUQDWLYH0HWKRG
2
n + 1
From (1), >1
an + b = pn + p n - 1
b = pn an + p ...(3) 2
from (2), ap + bnq = abn + ab ...(4) n 1 ! p
p ...(6)
Now from (3) and (4) n 1
aq
pn an + p =
an  a  nq
...(5) n 2  n a  b  1
apn2 a2n2 + apn + apn a2n + ap n2pq From (1) and (2) p b a
q ( n  1) 2
+ aqn2 npq = aq
n(n + 1) a ap (n+1) + npq (n+1) aq(n2 1)
2 2

n + - 2
a b
=0
p b a
dividing by (n + 1) -1 =
Then na2 a [p (n + 1) + q(n - 1)] + npq = 0 q (n + 1) 2

Since 'a' is real, the discriminant


2
B2 4AC t 0 n a b
= 2 b
- >0
[p (n + 1) + q(n - 1)]2 4n2 pq t 0 (n + 1) a
p2 (n + 1)2 + q2 (n 1)2 + 2pq (n2 1) 4n2pq t 0
p2 (n + 1)2 + q2 (n 1)2 + 2pq (n2 1 2n2) t 0 p>q ...(7)
p2 (n + 1)2 + q2 (n 1)2 2pq (n2 + 1) t 0 From (6) and (7)
p2 (n+1)2 + q2 (n1)2 pq {(n + 1)2 + (n 1)2} t 0 n 1
2

(n + 1)2 p (p q) + (n 1)2 q (q p) t 0 q<p<p


n 1
(p q) {(n + 1)2 p (n 1)2 q} t 0
2
n  1 2 n 1
Hence q cannot lie between p and p .
(p q) n  1 p  q t 0 n 1

 Comprehensive Algebra

 If between any two quantities, there be inserted


Practice Problems 2n 1 arithmetic, geometric and harmonic
 If a be the first term and l be the last term of an means, show that nth means are in G.P.
A.P. and if one arithmetic mean be inserted
between every two successive terms of the A.P.,
prove that the sum of the original A.P. exceeds 3.9 ARITHMETICO-GEOMETRIC
1 PROGRESSION
the sum of the arithmetic means by (a + l).
2 A sequence each term of which is formed by
 Prove that the sum of 10 arithmetic means multiplying the corresponding terms of an A.P. and
between 8 and 30 is 190. a G.P. is called Arithmetico-Geometric Progression.
 Find the arithmetic progression in which the )RUH[DPSOH
arithmetic mean of the first n terms is equal to 1 + 3x + 5x2 + 7x3 + ..... is an A.G.P.
their number for any n. Here 1, 3, 5, .... are in A.P. and 1, x, x2, x3 ..... are
in G.P.
 If the arithmetic mean between a and b is twice
as great as the geometric mean, show that In general, let us consider
a:b=2+ 3 :2 3. the A.P. a , a + d , a + 2 d , ......., a + (n  1)d
and the G.P. 1, r, r2, ..............., rn1
 If odd number of G.M.s are inserted between
then the A.G.P. is
two given quantities a and b, show that the
a, (a+d)r, (a+2d)r2, ......, [a + (n  1)d] rn1
middle
G.M. = ab The general term or the nth term of an A.G.P. can
7 be obtained by multiplying the nth term of an A.P. &
 Insert 6 geometric means between 14 and . nth term of a G.P.
64
 Prove that the product of 5 geometric means It is t n = [a + (n  1)d] rn1
between 2 and 8 is 1024. For instance, the nth term of the series
3 2x + 4x2 + 6x3 + 8x4 + ........
 If 12 and 9 are the geometric and harmonic
5 is {2 + (n 1)2} . (x. xn 1) = 2n.xn
means, respectively, between two numbers, find
them. 6XPRIQWHUPVRIDQ$ULWKPHWLFR
 If the harmonic mean between two quantities is *HRPHWULF6HULHV
to their geometric means as 12 to 13, prove
Let Sn = a + (a + d) r + a (a + 2d) r2 + ....
that the quantities are in the ratio of 4 to 9.
+ [a + (n 1) d] rn 1 ...(1)
 If b is the harmonic mean between a and c, prove Multiplying both sides of (1) by common ratio r
1  1 11 rSn = 0 + ar + (a + d) r2 + ...... + [a + (n 1) d] rn
that .
b a b c a c ...(2)
a n 1  b n1 Subtracting (2) from (1), we get
 Prove that is the GM or HM between Sn (1r) = a + dr + dr2 + .... + drn 1] [a + (n1) d] rn
an  bn
1 Sn (1r) = a + d [r + r2 + .... + rn - 1] [a + (n1) d] rn
positive numbers a and b according as n = or
2 dr(1  r n 1 )
1. =a+ [a + (n 1) d] rn
 If 9 AMs and again 9 HMs be inserted between (1  r )
n 1 n
2 and 3 then prove that A +
6
= 5, A is any AM a  dr(1  r )  [a  ( n  1)d]r
H Sn = ,
1 r (1  r ) (1  r )
and H the corresponding HM. rz1 ...(3)
Sequence and Series 

6XPWR,QILQLW\
2 n 1  2  n 2 n 1  2  n
Sn = 2 =
If |r| < 1, i.e., 1 < r < 1 and n of then Lim rn = 0 2 n 1 2n
n of

a dr
? Sf = 1  r  (1  r ) 2 . If nof Sf = Lim 2  1n 1  nn = 2
n of
2 2
([DPSOH If the sum to infinity of the series
([DPSOH If positive square root of,
35 1 1 1 8
1 + 4x + 7x2 + 10x3 + .... is then find x. 1
a a . ( 2a) 2 a . ( 4a) 4 a . ( 8a)8a ..... f is ,
16 27
6ROXWLRQ Let Sf = 1 + 4x + 7x2 + 10x3 + ... f find the value of a.
...(1) 6ROXWLRQ
? x Sf = 0 + x + 4x2 + 7x3 + .... f ...(2) 1 1 1 1  1  1 ......f
a  2a  4a ......f 2a 4a 8a 8
Subtracting (2) from (1), we get a
.2 =
27
(1 x) Sf = 1 + 3x + 3x2 + 3x3 + ..... f
1 1 1 1 2
3x now   2 ........f =
(1 x)Sf = 1 + a 2 2 a
1 x
2
35 1  2x and 1  2  3  ..... f =
(1 x) 2a 4a 8a a
16 1  x
35(1 x)2 = 16 + 32x ( using sum of A.G.P.)
3
32x2 102x+ 19 = 0 1 1 8 1 1
(7x 19) (5x 1) = 0 ? aa . 2a
= = . 23 a =
27 3 3
19 ([DPSOH Prove the identity :
xz ( for infinite series |x| < 1)
7
1 1
1 (xn1 + n 1 ) + 2(xn 2 + n 2 )
x= . x x
5
2
([DPSOH Find the sum to n terms of the series 1 1 xn 1
+ ..... + (n 1) (x + ) + n = n 1 .
x x x 1
1  2  3  4  5  ........
2 4 8 16 32 6ROXWLRQ The sum on the left may be rewritten
as follows
Also find the sum (if it exists) if n o f.
6ROXWLRQ 1 2 n 1
n 1  n 2  .....  + [xn 1 + 2xn 2
x x x
1  2  3  4  5  .....  n
S= ...(1) + ....+(n 1) x] + n
2 4 8 16 32 2n
The first bracketed expression is equal to
S 1  2  3  .....  n  1  n
= ...(2) 1
2 4 8 16 2n 2 n 1 [ x + 2x2 + ....+ (n 1) xn 1]
xn
S 1  1  1  1  ........  1  n
= x[(n  1)x n  nx n 1  1]
2 2 22 23 2 4 2 n 2 n 1 =
x n (x  1) 2
1 1  1 The second bracketed expression is obtained from

2 2n n 1 n the first one by replacing x by 1/x.
= 1 n 1 = 1  n  n 1 Hence, we get the required result.
1 2 2 2
2
 Comprehensive Algebra

3URSHUWLHVRI6
Practice Problems
n
 Sum 1 + 3x + 6x2 + 10x3 + .... toinfinity, xbeing < 1. L k = nk, where k is a constant .
r 1
n n
 Find 6 u n if un = 6 1n k
n 02
n 1 2 = 2 + 2 +...... to k terms = 2k
i 1
1 1 1
 Sum a + , 3a , 5a + + .... to 2p terms. n n
3 6 12
LL k ar = k a r,
 Determine the sums of the following series r 1 r 1
(constant can be taken outside the 6 notation)
(i) 1 + 3  5  7  .....  2nn
1
1;
2 4 8 2 k k

3  5  7  .....  ( 1) n 1 2n  1 2r = 2 r
(ii) 1 . r 1 r 1
2 4 8 2 n 1 n n n
LLL (a r br) = ar br ,
3.10 SUMMATION OF SERIES r 1 r 1 r 1
6 notation can be separated in case of + and
Series are often represented in compact form, called signs.
sigma notation, using the Greek letter 6 (sigma) as n n n
means of indicating the summation involved. (r 2  r) = r 2  r
r 1 r 1 r 1
The sum of the first n terms of a series is represented by
n 6XPRISRZHUVRIQDWXUDOQXPEHUV
6 a i = a1 + a2 + a3 + a4 + ...... + an
i 1 D 6XPRIILUVWQQDWXUDOQXPEHUV
Q
where i is the index of summation, 1 is the lower
limit of summation and n is the upper limit of
r = 1+ 2 + 3 + .... + n
U 
summation. (sum using formula of A.P.)
Q n (n  1)
)RUH[DPSOH r=
U  2
n 5
E 6XPRIVTXDUHVRIILUVWQQDWXUDOQXPEHUV
(i) i stands for 1 + 2 + 3 + 4 + 5
i 1 Q

5
r= 12 + 22 + 32 + ..... + n2
n 5 5 U 
i is also written as n or n Q
i 1 n 1 1 n(n  1)(2n  1)
3
r = 6
U
(ii) (3r  2) means (3.1 + 2) + (3.2 + 2) + (3.3 + 2)
r 1 3URRI
4 We have (x + 1)2 x3 = 3x2 + 3x + 1
(iii) n2 stands for 12 + 22 + 32 + 42 Putting x = 1, 2, 3, 4,...., n then
1

3 23 13 = 3.12 + 3.1 + 1
r
(iv) r+2 means
1  2  3
1 2 2  2 3  2
32 23 = 3.22 + 3.2 + 1
1 43 33 = 3.32 + 3.3 + 1
Sequence and Series 

53 43 = 3.42 + 3.4 + 1 G 6XPRIIRXUWKSRZHURIILUVWQQDWXUDOQXPEHUV


.................................... n
(n + 1)3 n3 = 3.n2 + 3.n + 1 r4 = 14 + 24 + 34 + .... + n4
adding all of them, we get r 1

(n + 1)3 13 = 3 (12 + 22 + 32 + .... + n2) n


+ 3(1 + 2 + 3 + .... + n) + (1 + 1 + 1 +...+ n times) r4 can be found by using the identity
n3 + 3n2 + 3n = 6n2 + 36n + n r 1

n( n  1) k5 (k 1)5 = 5k4 10k3 + 10k2 5k + 1


? 36n2 = n3 + 3n2 + 3n 3 n
2 n
which gives n5 = 6 {r 5  ( r  1) 5}
Q n(n  1)(2n  1) r 1
r  6n2 =
U  6 n n n n n
= 5 6 r 4  10 6 r 3  10 6 r 2  5 6 r  6 1
r 1 r 1 r 1 r 1 r 1
F 6XPRIFXEHVRIILUVWQQDWXUDOQXPEHUV
n
After simplifying we get
r3 = 13 + 23 + 33 + .... + n3 n n
r 1 r4 =
30
(n + 1) (2n + 1) (3n + 3n  1)
2 r 1
n
n 2 (n  1) 2 n
3
r = = r
r 1 4 r = 1
3URRI
If nth term of a sequence is given by
We have
(x + 1)4 x4 = 4x3 + 6x2 + 4x + 1 tn = an3 + bn2 + cn + d
Putting x = 1, 2, 3, ...., n then when a, b, c, d are constants,
24 14 = 4.13 + 6.12 + 4.1 + 1 Then sum of n terms of
34 24 = 4.23 + 6.22 + 4.2 + 1 Sn = 6 tn = a 6 n3 + b 6 n2 + c 6 n + 6d
44 34 = 4.33 + 6.32 + 4.3 + 1
This can be evaluated using the above results.
..............................................
(n + 1)4 n4 = 4.n3 + 6.n2 + 4.n + 1 H 6XPRISURGXFWVRIQXPEHUVWDNLQJWZRDWDWLPH
adding all we get Given a set of n numbers a1 , a2 , a3 , .... , an , the
(n + 1)4 14 = 4(13 + 23 + 32 + .... + n3) sum of products of numbers taking two at a
+ 4(12 + 22 + 3 + ..... + n2) + 4(1 + 2 + 3 + ..... + n) time is represented by 6 a i a j
1d i  jd n
+ (1 + 1 + 1 + .... + n times)
We have (a1+a2 + a3 + .... + an)2
n4 + 4n3 + 6n2 + 4n = 46n3 + 56n2 + 46n + n
n
6.n( n  1)(2 n  1) 4.n( n  1) = 6 a 2i  2 6 a i a j
= 4 6n3 +  +n i 1 1d i  jd n
6 2 i.e., (sum of given numbers)2 = sum of squares
46 n2 = n4 + 4n3 + 6n2 of numbers + 2 (sum of products of numbers
+ 4n n (n + 1) (2n + 1) 2n (n + 1) n taking two at a time)
= n2 (n + 1)2 This formula is useful to find the sum of products
2 of numbers taking two at a time if the sum of
n n (n  1)
? r3 = 6n3 =
2
the numbers and the sum of their squares can
r 1 be obtained.
 Comprehensive Algebra

([DPSOH Find the sum of all the even positive n( 4n 2  1)


integers less than 200 which are not divisible by 6 . =
3
6ROXWLRQ The required sum = (2 + 4) + (8 + 10)
([DPSOH Find
+ (14 + 16) + .... + (194 + 196)
(33 23) + (53 43) + (73 63) + .... to 10 terms.
= 1 6 + 3 6 + 5 6 + .... + 65 6
= 6(1 + 3 + 5 + .... + 65) 6ROXWLRQ Let
Sn=(33 23) + (53 43) + (73 63) + ..... to n terms
32
=6 (1 + 65) = 6 33 33 = 6534 ... (1)
2
Now nth term of the sequence 3, 5, 7....
([DPSOH Sum to n terms the series whose nth = 3 + (n 1)2 = 2n + 1
term is 2n 1 + 8n3 6n2. ? nth term of the series 33 + 53 + ... is (2n + 1)3
6ROXWLRQ Let the sum be denoted by S; then nth term of the sequence 2, 4, 6, ... is
S = 62n - 1 + 8 6 n3 6 6n2 2 + (n1)2=2n
n 8n 2 (n  1) 2 6n( n  1)(2n  1) ? nth term of the series 23 + 43 + .... is (2n)3 = 8n3
= 2 1  
2 1 4 6 From (1), nth term of the given series tn = (2n + 1)3 8n3
= 2n 1 + n(n + 1) [2n (n + 1) (2n + 1)] = 8n3 + 3. 4 n2 . 1 + 3. 2n . 12 + 13 8n3
= 2n 1 + n(n + 1) (2n2 1). = 12n2 + 6n + 1
([DPSOH Find the sum to n terms of the series ? Sn = 612n2 + 66n + 61 = 126n2 +66n + n
1 + (1 + 2) + (1 + 2 + 3) + ..... 12 n (n  1)(2 n  1) 6 n( n  1)
6ROXWLRQ rth term of the given series = sum of = + n
6 2
numbers in the rth bracket = 2n(n + 1) 2n + 1) + 3n(n + 1) + n
tr = (1 + 2 + 3 + 4 + ..... + r) = n[2(2n2 + 3n + 1) + 3n + 3 + 1]
r ( r  1) 1 = n(4n2 + 9n + 6) = 4n3 + 9n2 + 6n
= = (r2 + r)
2 2 Putting n = 10, we get
n
1n n S10 = 4. 103 + 9.102 + 6. 10 = 4960.
? Sn = t r = r 2  r
r 1 2 r 1 r 1
 ([DPSOH  Determine the sums of the series

1 2 + 3 4 + ........... + (1)n 1 n ;
1 n (n  1)(2 n  1)  n ( n  1)
= 6ROXWLRQ First assume that n is even. Put n = 2m.
2 6 2
Then 1 2 + 3 4 + .... + (1)n 1n
1 n(n  1) 2 n  1  1
= . = 1 2 + 3 4 + .... + (2m 1) 2m
2 2 3
n
n( n  1) 2 n  4 n( n  1)(n  2) = (1+3 + ... + 2m1) (2 + 4 + ... +2m) = m= .
= 2
4 3 6 Now let n be odd and put n = 2m 1. Then our sum
([DPSOH Find the sum 12+32+52 + ... + (2n 1)2. takes the form
6ROXWLRQ We can write the sum in the form [12 + 34 + ... (2m 2)] + (2m 1) = (m1)
Sn = {12 + 22 + 32 + 42 + ... + (2n 1)2 n 1
+ 2m 1 = m =
+ (2n)2} {22 + 42 + ... + (2n)2} 2
2n n Thus, if we put
= 6 k2  4 6 k2 1 2 + 3 4 + .... + (1)n 1n = S,
k 1 k 1
n n 1
2 n(2 n  1)( 4 n  1) 4 n( n  1)(2 n  1) then S = if n is even, S = if n is odd.
= 2 2
6 6
Sequence and Series 

However, this result can be obtained in a simpler 1 1 1


... n, ... and whose common ratios are , , ,
way. Indeed, if n is even, we have 2 3 4
S = [1 2] + [3 4] + [5 6] + .... 1 2 n 1
2
..., , ... respectively, then find the value of Sr .
n n 1 r 1
+ [(2m 1) 2m] = 1 . m = m = 6ROXWLRQ rth series will have a = r and common
2
Hence we also get the result for odd n. 1
ratio =
r 1
([DPSOH Find the sum of the series
r ( r  1)
12 22 + 32 + .... + (1)n + 1 n2. ? Sr = r = = r + 1 ? 6U = (r + 1)2
1 1 r
6ROXWLRQ We consider two cases, n odd and n
r 1
even. Suppose n is odd so that n = 2m + 1 for some 2 n 1 2 n 1
m. In this case the sum Sn is given by ? S2r = ( r  1) 2 = 22 + 32 + 42 + .......
Sn = 12 22 + 32 42 + ... + (2m + 1)2 r 1 r 1

= {12 + 22 + 32 + 42 + ... + (2m)2 +(2n 1)2 + (2n)2


+ (2m + 1)2} 2{22 + 42 + ... + (2m)2} = 12 + 22 + 32 + ....... + (2n)2 1
= sum of the square of the first (2n) natural
2 m 1 m numbers
= k2 8 k
2

k 1 k 1 (2 n )(2 n  1)( 4 n  1)
= 1
(2m  1)(2 m  2 )( 4m  3) 6
=
6 n (2 n  1)( 4 n  1)
8m (m  1)(2 m  1) = 1
3
6 ([DPSOH  Find the sum to n terms of the
(2m  1)(2 m  2) n (n  1) series,
? Sn = = 2
2 0.7 + 7.7 + 0.77 + 77.7 + 0.777 + 777.7
If n is even. then n = 2m for some m. Hence the sum + 0.7777 + ...... where n is even .
Sn is given by 6ROXWLRQ n = 2m
S = (0.7 + 0.77 + 0.777 + ...... m term)
Sn = 12 22 + 32 42 + ... + (2m 1)2 (2m)2 + (7.7 + 77.7 + 777.7 + ...... m terms)
= {12 + 22 + 32 + 42 + ... + (2m 1)2
7
+ (2m)2} 2{22 + 42 + ... + (2m)2} = (0.9 + 0.99 + 0.999 + ..... m terms)
9
2m m
= k2  8 k2 +
7
((102  1) + (103  1) + ..... + (10m + 1  1))
k 1 k 1
90
2m (2 m  1)( 4m  1)
 7 n 1 1
6  1  n /2
8m (m  1)(2 m  1) 9 2 9 10

6 7 100 n
= m(2m + 1) =
 n( n  1)
+
90 9

10 n / 2  1 
2

2
( 1) n 1 n( n  1) ([DPSOH Find the number of shots arranged
Thus for any n, Sn = in a complete pyramid the base of which is an
2
equilateral triangle, each side containing n shots.
([DPSOH If S1, S2, S3, ... Sn, .... are the sums of 6ROXWLRQ Suppose that each side of the base
infinite geometric series whose first terms are 1, 2, 3, contains n shots; then the number of shots in the lowest
layer is
 Comprehensive Algebra

n + ( n 1) + (n 2) + ....... + 1 Also a1 = 5, a2 = 7 ...(ii)


We are required to find an and a1 + a2 + a3 + ... + an.
n ( n  1) 1 ( n 2  n ). Substituting k = 2, 3, ..... , n successively in (i)
that is, =
2 2 throughout, we have
In this result write n 1, n 2,.... for n , and we thus a2 = a1 + 2,
obtain the number of shots in the 2nd, 3rd,.... layers. a3 = a2 + 3,

1 n ( n  1)( n  2)
? S= (6n2 + 6n) = 
2 6
an = an1 + n
([DPSOH There are n necklaces such that the
Adding column-wise and simplifying, we get
first necklace contains 5 beads, the second contains
an = a1 + (2 + 3 + .... + n), ...(iii)
7 beads and, in general the ith necklace contains i
The terms a2, a3 ..... , an1 cancelling out on both
beads more than the number of beads in (i 1)th
sides.
necklace. Find the total number of beads is all the n
From (iii), we have
necklaces.
6ROXWLRQ Let us write the sequence of the number 1 1
an = 5 + n(n + 1) 1, = (n2 + n + 8).
of beads in the 1st, 2nd, 3rd, ...., nth necklaces. 2 2
= 5, 7, 10, 14, 19, ........ Since

{ }
= (4 + 1), (4 + 3), (4 + 6) (4 + 10), (4 + 15),. ......,
1 1 1
Sa n = n(n + 1) (2n + 1) + n(n + 1) + 8n ,
n (n  1) 2 6 2
4  2
1
Sn = Total number of beads in the n necklaces = n(n 2 + 3n + 26)
6
which is the total number of beads in all the n
Sn = 4 
4

 ....
 4 + 1 + 3 + 6 + .... necklaces.
n times
([DPSOH The squares of the natural numbers
n (n  1) are grouped like (12); (22, 32, 42); (52, 62, 72, 82, 92);
+ and so on. Find the sum of the elements in nth group.
2
= 4n + Sum of the first n triangular numbers. 6ROXWLRQ By observations, the last element of the
nth group = n4
1 1
= 4n + (6n2 + n) = 4n + (6n2 + 6n) The number of elements in nth group = (2n 1)
2 2
The first element of the nth group = (n2 2n + 2)2
1 n (n  1)(2 n  1)  1 n( n  1) Hence sum of the numbers in nth group are
= 2n +
2 6 2 2 S = (n2 2n + 2)2 + (n 2 2n + 3)2 + ..... + (n2)2
n (n  1)(2 n  1) n (n  1) S = [12+22+32 + ..... (n2)2] [12+ 22+32+((n1)2)2]
= 4n + 
12 4 n 2 (n 2 + 1)(2n 2 + 1)
S= -
1 n 6
= [48n + 2n(n+1) (n+2)] = [n2 + 3n + 26].
12 6 (n - 1)2 (n 2 - 2n + 2)(2n2 - 4n + 3)
$OWHUQDWLYH 6
Let us denote the number of beads in the kth ([DPSOH Find the value of the expression
necklace by ak. j
n i
We are given that 1
ak = ak1 + k, for k t 2. ...(i) i 1 j 1k 1
Sequence and Series 

n i j n i
6ROXWLRQ  Show that
1 j
i 1 j 1k 1 i 1k 1 1 u 2 2  2 u 3 2  ...  n u (n  1) 2 3n  5
1 u 2 2  2 2 u 3  ...  n 2 u (n  1) 3n  1
n
i( i  1) 1 n 2 n
= = i  i
i 1 2 2 i 1 i 1  Find the sum of n terms of the series whose nth

term is 4n (n2 + 1) (6n2 + 1).
1 n( n  1)(2 n  1)  n( n  1)
 Find the sum of n terms of the series whose nth
2 6 2
term is 3(4n + 2n2) 4n3.
n (n  1) n ( n  1)( n  2)
= [2n + 1 + 3] =  Sum to n terms the series
12 6
1 . n2 + 2(n 1)2 + 3(n 2)2 + .....
([DPSOH The natural numbers are arranged
in groups as given below;  Show that whether n is odd or even,
1 12 22 + 32 42 + ....... to (n 1) terms
2 3 1
4 5 6 7 8 = (1)nn(n1).
2
9 10 11 12 13 14 15
............................................................................................  Find (i) the sum of the squares, (ii) the sum of
Prove that the sum of the numbers in the nth group the products taken two together of the
is 2n  2 {2n + 2n  1  1}. numbers 1, 4, 7, .... (3n 2).
6ROXWLRQ Note that nth group has 2n  1 terms .  Find the number of shots which can be arranged
1st term in the nth group is 2n  1 and the last term in in a pyramidal pile on a triangular base, each
the nth group is 2n  1 .
side of the base containing 10 shot.
? Sum of the terms in the nth group
N  If S1, S2, S3,......, Sp are the sums of infinite
= (A + L) ; N = 2n  1 ; A = 2n  1 ; geometric series, whose first terms are 1, 2,
2
3,....,p, and whose common ratios are
L = 2n  1 = 2n  2 {2n + 2n  1  1}
1 , 1 , 1 ,.... 1
2 3 4 p  1 respectively, prove thatt
Practice Problems p
S1 + S2 + S3 +... + Sp= (p +3).
 Sum the series 2
2.3 + 3.6 + 4.11 + .... + (n + 1)(n2 + 2)
3.11 RECURRING SERIES
 112 122 + 132 .... 202 + 212.
A series a0 + a1 + a2 + a3 + ..., in which from and after
 Find the sum 502 492 + 482 472 + .... + 22 1.
a certain term each term is equal to the sum of a fixed
 203 193 + 183 ... + 23 13. numbe r of the preceeding terms multiplied
 If S1, S2, S3 are the sum of first n natural numbers, respectively by certain constants is called a recurring
their squares and their cubes, respectively, show series.
that In the series 1 + 2x + 3x2 + 4x3 + 5x4 + ..., each term
9 S22 = S3(1 + 8S1) after the second is equal to the sum of the two
3.46 C o m p r e h e n s iv e A lg e b r a

p re c e e d in g te r m s m u ltip lie d re s p e c tiv e ly b y th e fo rm a n A .P . H o w e v e r th e ir s u c c e s s iv e d iffe re n c e s


c o n s ta n ts 2 x , a n d x 2 ; th e s e q u a n titie s b e in g c a lle d 5 2 1 2,
c o n s ta n ts b e c a u s e th e y a re th e s a m e fo r a ll v a lu e s o f 9 2 5 2 , 1 3 2 9 2 , .... fo rm a n A .P .
n . T h u s 5 x 4 = 2 x . 4 x 3 + ( x 2 ) . 3 x 2 ;
S n = 1 + 2 5 . 5 + 8 1 . 5 2 + 1 6 9 . 5 3 + ....
th a t is , a 4 = 2 x a 3 x 2 a 2 .
... + (4 n + 1 ) 2 . 5 n 1 ... (1 )
C o n s id e r th e s e rie s 5 .S n = 2 3
1 . 5 + 1 5 . 5 + 8 1 . 5 + ....
t 1 + t 2 x + t 3 x 2 + t 4 x 3 + t 5 x 4 + ........... ....(1 ) .... + (4 n 3 ) . 5 n 1 + (4 n + 1 ) 2 . 5 n ... (2 )
In th is ty p e o f s e rie s e v e ry te rm is a p ro d u c t o f tw o
F ro m (1 ) (2 )
n u m b e rs in w h ic h o n e fo rm a G .P .
x 0 , x 1 , x 2 , x 3 , x 4 ,...... fo rm a n G .P . in a b o v e 4 .5 5 6 .5 2 8 8 .5 3 ...( )
S n (1 5 ) = 1 + 2
s e rie s . n 1 te rm

Case I : If t1 , t2 , t3 , ......... fo rm a n A .P . (4 n + 1 ) 2 . 5 n ...(3 )


(th e n th e s e rie s (1 ) is c a lle d o n a rith m e tic o - g e o m e tric 5 ( 1 5 ) S n = 1 .5 + 2 4 . 5 2 + 5 6 . 5 3 + .....
s e rie s , d is c u s s e d b e fo re ) ... + 8 (4 n 1 ) 5 n (4 n + 1 ) 2 . 5 n + 1 ...(4 )
F ro m (3 ) (4 )
Case II : If t 1 , t 2 , t3 , t4 , ......... d o n o t fo rm a n A .P . b u t t 2
S n (1 5 ) 5 (1 . 5 ) S n = S n (1 5 ) 2
t 1 = T 1 , t 3 t 2 = T 2 , t 4 t 3 = T 3 , ..... fro m a n A .P .
M u ltip ly e q u a tio n (1 ) b y x , th e c o m m o n ra tio o f 3 2 .5 2 3 2 .5 3 .....
g e o m e tric s e rie s a n d s u b tra c t th is c u rre n t e q u a tio n = 1 + 2 3 . 5 +
( n 2 ) te rm
fro m e q u a tio n (1 ).
8 (4 n 1 ) 5 n + (4 n + 1 ) 2 . 5 n + 1 ... (5 )
Case III: T 1 , T 2 , T 3 , .... d o n o t fo rm a n A .P . B u t T 1 T 2 ,
3 2 .5 2 ( 5 n 2 1 )
T 3 T 2 , T 4 T 3 , .... fro m a n A .P . = 1 + 23.5 +
5 1
F o r e v a lu a tin g th e s e ty p e s e rie s y o u c a n m u ltip ly 8 (4 n 1 ) 5 n + (4 n + 1 ) 2 . 5 n + 1
th e la s t e q u a tio n (m e a n s th e e q u a tio n a t s ta g e (A ))
b y th e c o m m o n ra tio o f G .P . & s u b tra c t th is e q u a tio n ( 4 n 1 ) 2 .5 n 1 1 6 .5 n 4 0 n .5 n 8 4
Sn=
fro m (A ). T h is p ro c e s s is r e p e a te d u n itl y o u g e t th e (1 5 ) 2
fin a l e q u a tio n in G .P .
Example 1. F in d th e s u m o f 3.12 M ETH O D O F SU BTRA CTIO N
1 2 + 2 2 x + 3 2 x 2 + 4 2 x 3 + ...... to . If th e d iffe re n c e s o f th e s u c c e s s iv e te rm s o f a s e rie s
Solution L e t S = 1 2 + 2 2 x + 3 2 x 2 + 4 2 x 3 + ...... a re in A .P . o r G .P ., w e c a n fin d n th te rm o f th e s e rie s
(1 x )S = 1 + 3 x + 5 x 2 + 7 x 3 + ...... b y th e fo llo w in g s te p s :
(1 x ) 2 S = 1 + 2 x + 2 x 2 + 2 x 3 + ...... Step I : D e n o te th e n th te rm a n d th e s u m o f th e s e rie s
1 u p to n te rm s o f th e s e rie s b y T n a n d S n re s p e c tiv e ly .
= 1 + 2x . w h e n |x | < 1 ,
1 x Step II : R e w rite th e g iv e n se rie s w ith e a c h te rm s h ifte d
1 x 1 x b y o n e p la c e to th e rig h t.
= S = (1 x ) 3
1 x Step III : S u b tra c tin g th e a b o v e tw o fo rm s o f th e se rie s ,
Example 2. E v a lu a te 1 2 + 5 2 . 5 + 9 2 . 5 2 + 1 3 2 . 5 3 fin d T n .
+ ......... to n te rm s . L e t u 1 , u 2 , u 3 ...... b e a s e q u e n c e , s u c h th a t u 2 u 1 , u 3
Solution T h is s e r ie s is n o t a n a r ith m e tic o - u 2 , ............. is e ith e r a n A .P . o r a G .P . th e n n th te rm u n
g e o m e tric s e rie s , b e c a u s e 1 2 , 5 2 , 9 2 , 1 3 2 , ........ d o n o t o f th is s e q u e n c e is o b ta in e d a s fo llo w s

03-Chap_Sequence.p65 46 3/25/2013, 4:47 PM


Sequence and Series 

S = u1 + u2 + u3 + ........... + un ...(i) From (1) (2) ; 0 = 2 + 4 + 16 + 64 + 256 + ...


S = u1 + u2 + .......... + un 1 + un ...(ii) ........ + tn tn1 tn
(i) (ii) tn = 2 + 4+16+64+256 + .......tn tn1
un = u1 + (u2 u1) + (u3 u2) + ..... + (un un 1) (n1) terms are in A.P.

where the series (u2 u1) + (u3 u2) + .... + (un un 1)


is either in A.P. or in G.P. then we can find un and 4( 4 n 1  1) 1
k = 2+ = (2 + 4n)
n 1 3
hence sum of this series as S = ur
r 1
1 n 2 4
n n
&DVH If (t2 t1), (t3 t2), ....., (tn1 tn) are in A.P. ? Sn= t r = + r
r 1 3 i =1 i =1
e.g. the series
x, x + y, x + 2y + z, x + 3y + 3z, x + 4y + 6z, ... ; ( 4 n 1  3n 2  3n  4)
=
the differences of consecutive terms are y, y + z, y + 9
2z, y+ 3z, ....... which are in A.P. &DVH
It is not always necessary that the series of first order
([DPSOH Evaluate 1 + 5 + 12 + 22 + 35 + ............ of differences i.e. u2 u1, u3 u2 ...... un un 1 is
6ROXWLRQ Sn = 1 + 5 + 12 + 22 + 35 + ......... + tn ...(1) always either in A.P. or in G.P. In such case let
Sn = 1 + 5 + 12 + 22 + ......... + tn1 + tn ...(2) u1 = T1, u2 u1 = T2, u3 u2 = T3 .......,un un 1 = Tn.
So un = T1 + T2 + ....... + Tn .... (i)
From (1) (2); 0 = 1 + 4 + 7 + 10 + 13 + ......
un = T1 + T2 + ........ + Tn 1 + Tn .... (ii)
....... + tn tn 1 tn
(i) (ii)
tn = 1 + 4+7+10+13 + .......
Tn = T1 + (T2 T1) + (T3 T2) + ...... + (Tn + Tn 1)
(n1) terms are in A.P.

Now, the series (T2 T1) + (T3 T2) + ..... + (Tn Tn 1) is


n 1 3n 2  n
=1+ [2 4 + (n 1) 1) 3] = series of second order of differences and when it is
2 2
either in A.P. or in G.P., then un = u1 + 6 Tr
n n
2
? Sn = tr = 3r 2 r Otherwise in the similar way we find series of higher
order of differences and the nth term of the series.
r 1 r 1
1 3.n (n  1)(2 n  1) n( n  1) ([DPSOH Evaluate
= 
2 6 2 2 + 5 + 12 + 31 + 86 + ............. to n terms
6ROXWLRQ
n3  n2 S = 2 + 5 + 12 + 31 + 86 + ......... tn ... (1)
=
2 S = 2 + 5 + 12 + 31 + ......... tn1 + tn ... (2)
&DVH If t2 t1, t3 t2, ......, (tn 1 tn) are in G.P. From (1) (2) ;
For example the series 2 + 3 + 7 + 19 + 55 + ...... + tn tn1 tn
a, a + b, a + b + br, a + b + br + br2, ................ ; tn = 2 + 3 + 7 + 19 + 55 + ..... + ( tn tn 1) ...(3)
the differences of consecutive terms are b, br, tn = 2 + 3 + 7 + 19 + ..........
+ ( tn 1 tn 2 ) ( tn tn 1 ) ...(4)
br2............... which are in G.P.
__________________________________________________________
([DPSOH Evaluate 2 + 6 + 22 + 86 + 342 + 0 = 2 + 1+4+12+36 + ....... (....) Tn
.......... to n terms (n2) term

6ROXWLRQ Sn = 2 + 6 + 22 + 86 + 342 + .... tn ... (1)


Tn = 2 + 1 + 4+12+36 + ............
Sn = 2 + 6 + 22 + 86 + ... + tn1 tn ... (2) n2 term
Tn = 2 + 1 + 4 (1 + 3 + 32 + ........ ) = 1 + 2 . 3n 2
 Comprehensive Algebra

6ROXWLRQ Let
n
2.(3n 1  1)
Now, tn = 2 + Tr =2+(n1) + S = 2 + 12 + 36 + 80 + 150 + 252 + ..... + Tn ... (i)
r 1 3 1
S = 2 + 12 + 36 + 80 + 150 + 252 + ....... + Tn 1 + Tn
= n + 3n1
... (ii)
n
(r + 3r -1 )
n
(i) (ii)
? Sn = t r =
r 1 r =1 Tn = 2 + 10 + 24 + 44 + 70 + 102 + .... + (Tn Tn 1)
n (n  1) 1(3n  1) 1 2 ... (iii)
= + = [n + n + 3n 1] Tn = 2 + 10 + 24 + 44 + 70 + 102 + .........
2 3 1 2
([DPSOH Find the sum to n terms + (Tn 1 Tn 2) + (Tn Tn 1) ... (iv)
3 + 7 + 13 + 21 + ...... (iii) (iv)
6ROXWLRQ Let Tn Tn 1 = 2 + 8 + 14 + 20 + 26 + .......
S = 3 + 7 + 13 + 21 + ........ + Tn ...(i)
S = 3 + 7 + 13 + ........+ Tn 1 + Tn ...(ii) n
= [4 + (n1) 6] = n [3n 1] = Tn Tn 1 = 3n2 n
(i) (ii) 2
Tn = 3 + 4 + 6 + 8 + ....... + (Tn Tn 1) ? The general term of given series is
n 1 6 Tn Tn 1 = 6 3n2 n = n3 + n2.
=3+ [8 + (n + (n 2)2] Hence sum of this series is
2
= 3 + (n 1) (n + 2) = n2 + n + 1
n 2 (n  1) 2 n( n  2)(2n  1)
Hence S = 6 n3 + 6n2 = 
S = 6(n2 + n + 1) = 6 n2 + 6 n + 61 4 6
n (n  1)(2 n  1) n (n  1) n (n  1)
=  +n = (3n2 + 7n + 2)
6 2 12
Q 1
= (n2 + 3n + 5) n(n + 1) (n + 2) (3n + 1)
 12
([DPSOH Find the sum of n-terms ([DPSOH Find the general term and sum of n
1 + 4 + 10 + 22 + ....... terms of the series 9, 16, 29, 54, 103
6ROXWLRQ Let 6ROXWLRQ Let
S = 1 + 4 + 10 + 22 + ...... + Tn ... (i) S = 9 + 16 + 29 + 54 + 103 + .... + Tn ... (i)
S = 1 + 4 + 10 + ........+Tn 1 + Tn ... (ii) S = 9 + 16 + 29 + 54 + 103 + .... + Tn 1 + Tn ... (ii)
(i) (ii) Tn = 1 + (3 + 6 + 12 + ... + Tn Tn 1) (i) (ii)
Tn = 9 + 7 + 13 + 25 + 49 + .... + (Tn Tn 1)... (iii)
2 n 1  1 Tn = 9 + 7 + 13 + 25 + 49 + ........ +
Tn = 1 + 3
2 1 (Tn1 Tn 2) + (Tn Tn 1) ...(iv)
T n = 3 . 2n 1 2 (iii) (iv)
2n 1 6  12 
24  .......
So S = 6 Tn = 3 6 2n 1 62 = 3 . 2n Tn Tn 1 = 9 + (2) + ( n 2 ) terms

2 1
= 3.2n 2n 3 = 7 + 6 [2n 2 1] = 6(2)n 2 +1.

([DPSOH Find the nth term and the sum of n ? The general term is Tn = 6(2)n 1 + n + 2
Also sum S = 6Tn = 662n 1 + 6n + 62
term of the series 2, 12, 36, 80, 150, 252.
Sequence and Series 

(2 n  1) n(n  1)  2, 5, 12, 31, 86,.....


2 3
= 6.   2 n + 2n = 6(2n 1)  Let x = 1 + 3a + 6a + 10a + .........., |a| < 1;
2 1 2 2 3
y = 1 + 4b + 10b + 20 b + ........., |b| < 1.
n (n  5) 2
Find S = 1 + 3 (ab) + 5 (ab) + .... in terms of x
+
2 and y.
([DPSOH  The nth term, an of a sequence of
numbers is given by the formula an = an 1 + 2n for n t 3.13 METHOD OF DIFFERENCES
2 and a1 = 1. Find an equation expressing an as a
polynomial in n. Also find the sum to n terms of the If the rth term of a series can be expressed as the
sequence. difference of two quantities one which is the same
function r that the other is of r 1, the sum of the
6ROXWLRQ a1 = 1 series may be readily found.
a2 = a1 + 4 = 1 + 4 = 5 For let the series be denoted by t1 + t2 + t3 + ..... + tn,
a3 = a2 + 6 = 5 + 6 = 11 and its sum by Sn, and suppose that any term tr can
a4 = a3 + 8 = 11 + 8 = 19 and so on hence be put in the form vr vr 1; then
S = 1 + 5 + 11 + 19 + ........ + an Sn = (v1 v0) + ( v2 v1) + .... + (vn 1 vn 2)
S = + 1 + 5 + 11 + ........ + an 1 + an + (vn vn 1) = vn v0.
( ) In other words, if the nth term of a series can be written
0 = 1 + 4 + 6 + 8 + ........ + (an an 1) an
as tn = f(n) f(n 1)
an = 1 + 2 2  3  4  .......  (a n  a n 1 )

n n n
( n 1) terms then Sn = 6 t n = 6 f (n ) 6 f ( n r 1)
n 1 n 1 n 1
n 1 = {f(1) + f(2) + ..... + f(n)} {f(2) + f(3)
an = 1 + 2 [4 + (n 2)] = 1 + (n 1) (n + 2)
2 + .... + f(n + 1)} when + sign is taken
= n2 + n 1 .
or {f(1) + f(2) + .... + f(n) } {f(0) + f(1) + ... + f(n+ 1)}
sum = an = n2 + n 1 when sign is taken
n ( n  1)(2 n  1) n (n  1) = f(1) f(n + 1) when + sign is taken
= + n or f(n) f(0) when sign is taken.
6 2
n ([DPSOH Find the sum to n-terms of the series
[(n + 1)(2n + 1) + 3(n + 1) 6]
6 1.2 + 2.3 + 3.4 + ........
n
= [2n2 + 3n + 1 + 3n + 3 6] 6ROXWLRQ Let Tr be the general term of the series
6
So Tr = r(r + 1)
n n( n 2  3n  1)
= [2n2 + 6n 2] = To express tr = f(r) f(r 1) multiply and divide tr by
6 3
[(r + 2) (r 1)]
r
Practice Problems So Tr = (r + 1) [(r + 2) (r 1)]
3
)LQGWKHQWKWHUPDQGWKHVXPRIQWHUPVRI 1
= [r (r + 1) (r + 2) (r 1) r (r + 1)].
WKH VHULHV 3
1
 2 + 7x + 25x2 + 91x3 + .... Let f(r) = r(r + 1) (r + 2)
3
 1 + 2x + 6x2 + 20x3 + 66x4 + 212x5 + ....
so Tr = [f(r) f(r 1)].
 1 + 3 + 7 + 15 + 31 + ..... n
 2, 12, 36, 80, 150, 252,.... Now S = Tr = T1 + T2 + T3 + ...... + Tn
 9, 16, 29, 54, 103,.... r 1
 Comprehensive Algebra

1 1 1  1  1  1  1  1
T1 = [1 . 2 . 3 0] ? sn =
3 3 2 3 n  1 n  2 n  3
1
T2 = [2 . 3 . 4 1 . 2 . 3] 11  1 1  1  1 ;
3 =
1 18 3 n  1 n  2 n  3
T3 = [3 . 4. 5 2 . 3 . 4]
3 ([DPSOH Sum to n terms of the series
 1 1 1
1 + + + ...
Tn = [n(n + 1) (n + 2) (n 1)n (n + 1)] (1 + x)(1 + 2x) (1 + 2x)(1 + 3x) (1 + 3x)(1 + 4x)
3
6ROXWLRQ Let Tr be the general term of the series
1
? S = n (n + 1) (n + 2) 1
3 Tr = (1  rx )(1  (r  1)x )
Hence sum of series is f(n) f(0).
([DPSOH Sum the following series upto n-terms 1 [1  (r  1)x ]  (1  rx )
x (1  rx )(1  (r  1)x )
So Tr =
1 2 3 4 + 2 3 4 5 + 3 4 5 6 + .............
6ROXWLRQ
1 1  1
Sn = 1 2 3 4 + 2 3 4 5 + 3 4 5 6 + ........
x 1  rx 1  (r  1)x

=
1
Tn = n(n + 1)(n + 2)(n + 3) [(n + 4) (n 1)] Tr = f(r) f(r + 1)
5
? S = 6 Tr = T1 + T2 + T3 + ........ + Tn
1
? T1 = [1 2 3 4 5 0]
5 1 1  1
x 1  rx 1  ( n  1)x

=
1
T2 = [2 3 4 5 6 1 2 3 4 5]
5 n
............................................................ = (1  x )[1  ( n  1)x ]
1
Tn = [n(n + 1)(n + 2)(n + 3)(n + 4) ([DPSOH Prove that
5
(n 1)n(n + 1)(n + 2)(n + 3)] 3 5 7 9
+ + + + ....
1 12 12 + 22 12 + 22 + 32 12 + 22 + 32 + 4 2
? Sn = Tn = [n(n + 1)(n + 2)(n + 3)(n + 4)]
5 6n
to n terms =
n 1
1 6ROXWLRQ Also find the sum to infinite terms of
Sn = n(n + 1)(n + 2)(n + 3)(n + 4).
5
the series on the left hand side.
([DPSOH Sum the series 3  ( n  1)2
1  1  1  .....  1 tn = 2
1  2 2  32  ....  n 2
1 .4 2 .5 3 .6 n (n  3) 2n  1
= n ( n  1)( 2 n  1) / 6
1 1  1
6ROXWLRQ un = ;
3 n n 3 6 1 1
= n (n  1) = 6 
1  1  1  .....  1  1  1  1  ... n n  1
? 3sn =
1 2 3 n 4 5 6 1 1 1
? t1 = 6 1  , t2 = 6  ,
1  1  1 2 2 3
n 1 n  2 n  3 1 1
t3 = 6  , ................ , tn = 6 1  1
3 4 n n 1

You might also like